Sunteți pe pagina 1din 231

MATHEMATICS

CONTENTS

1. Sample paper (By CBSE).…….…...………………………………………………..….01-17

2. Sample paper 01………….……………………………………………………………..…18-31

3. Sample paper 02………….……………………………………………………………..…32-45

4. Sample paper 03………….……………………………………………………………..…46-65

5. Sample paper 04………….……………………………………………………………..…66-81

6. Sample paper 05………….……………………………………………………………..…82-96

7. Sample paper 06………….……………………………………………………………..…97-110

8. Sample paper 07………….……………………………………………………………..…111-135

9. Sample paper 08………….……………………………………………………………..…136-150

10. Sample paper 09………….……………………………………………………………..…151-167

11. Sample paper 10………….……………………………………………………………..…168-186

12. Sample paper 11………….……………………………………………………………..…187-212

13. Sample paper 12………….……………………………………………………………..…213-229


CBSE Class-X Mathematics
Sample Paper (By CBSE)

General Instructions:

1. All questions are compulsory.


2. The question paper consists of 30 questions divided into four sections A, B, C and D.
3. Section A contains 6 questions of 1 mark each. Section B contains 6 questions of 2 marks
each. Section C contains 10 questions of 3 marks each. Section D contains 8 questions of 4
marks each.
4. There is no overall choice. However, an internal choice has been provided in four
questions of 3 marks each and three questions of 4 marks each. You have to attempt only
one of the alternatives in all such questions.
5. Use of calculators is not permitted.

Section A
Question numbers 1 to 6 carry 1 mark each

1. Write whether the rational number will have a terminating decimal expansion or a
nor-terminating repeating decimal expansion.
2. Find the value(s) of k, if the quadratic equation has equal roots.
3. Find the eleventh term from the last term of the AP:
27, 23, 19, ..., –65.
4. Find the coordinates of the point on y-axis which is nearest to the point (–2, 5).
5. In given figure, and Find the ratio of the area of
to the area of

6. If find the value of

Material downloaded from myCBSEguide.com. 1 / 17


1
Section B
Question numbers 7 to 12 carry 2 marks each.

7. If two positive integers p and q are written as are prime


numbers, then verify:

8. The sum of first n terms of an AP is given by Find the sixteenth term of


the AP.
9. Find the value(s) of k for which the pair of linear equations
have infinitely many solutions.

10. If is the mid-point of the line segment joining the points (2, 0) and then

show that the line passes through the point


11. A box contains cards numbered 11 to 123. A card is drawn at random from the box. Find
the probability that the number on the drawn card is
i. a square number
ii. a multiple of 7
12. A box contains 12 balls of which some are red in colour. If 6 more red balls are put in the
box and a ball is drawn at random, the probability of drawing a red ball doubles than
what it was before. Find the number of red balls in the bag.

Section C
Question numbers 13 to 22 carry 3 marks each.

13. Show that exactly one of the numbers is divisible by 3.


14. Find all the zeroes of the polynomial if two of its zeroes are

15. Seven times a two digit number is equal to four times the number obtained by reversing
the order of its digits. If the difference of the digits is 3, determine the number.
16. In what ratio does the x-axis divide the line segment joining the points
Find the co-ordinates of the point of division.

OR

The points form a parallelogram. Find the


length of the altitude of the parallelogram on the base AB.

Material downloaded from myCBSEguide.com. 2 / 17


2
17. In given figure then prove that

OR

In an equilateral triangle ABC, D is a point on the side BC such that Prove


that

18. In given figure are two parallel tangents to a circle with centre O and
another tangent AB with point of contact C intersecting at A and at B. Prove
that

19. Evaluate:

OR

If then evaluate:
20. In given figure ABPC is a quadrant of a circle of radius 14 cm and a semicircle is drawn
with BC as diameter. Find the area of the shaded region.

Material downloaded from myCBSEguide.com. 3 / 17


3
21. Water in a canal, 6 m wide and 1.5 m deep, is flowing with a speed of 10 km/h. How much
area will it irrigate in 30 minutes, if 8 cm of standing water is needed?

OR

A cone of maximum size is carved out from a cube of edge 14 cm. Find the surface area of
the remaining solid after the cone is carved out.
22. Find the mode of the following distribution of marks obtained by the students in an
examination:

Marks obtained 0-20 20-40 40-60 60-80 80-100

Number of students 15 18 21 29 17

Given the mean of the above distribution is 53, using empirical relationship estimate the
value of its median.

Section D
Question numbers 23 to 30 carry 4 marks each.

23. A train travelling at a uniform speed for 360 km would have taken 48 minutes less to
travel the same distance if its speed were 5 km/hour more. Find the original speed of the
train.

OR

Check whether the equation has real roots and if it has, find them by
the method of completing the square. Also verify that roots obtained satisfy the given
equation.
24. An AP consists of 37 terms. The sum of the three middle most terms is 225 and the sum of
the last three terms is 429. Find the AP.

Material downloaded from myCBSEguide.com. 4 / 17


4
25. Show that in a right triangle, the square of the hypotenuse is equal to the sum of the
squares of the other two sides.

OR

Prove that the ratio of the areas of two similar triangles is equal to the ratio of the
squares of their corresponding sides.
26. Draw a triangle ABC with side Then, construct a
triangle whose sides are times the corresponding sides of
27. Prove that
28. The angles of depression of the top and bottom of a building 50 metres high as observed
from the top of a tower are respectively. Find the height of the tower and
also the horizontal distance between the building and the tower.
29. Two dairy owners A and B sell flavoured milk filled to capacity in mugs of negligible
thickness, which are cylindrical in shape with a raised hemispherical bottom. The mugs
are 14 cm high and have diameter of 7 cm as shown in given figure. Both A and B sell
flavoured milk at the rate of per litre. The dairy owner A uses the formula
to find the volume of milk in the mug and charges for it. The dairy owner B is
of the view that the price of actual quantity of milk should be charged. What according to
him should be the price of one mug of milk? Which value is exhibited by the dairy owner

B?

30. The following distribution shows the daily pocket allowance of children of a locality. The
mean pocket allowance is Find the missing frequency k.

Daily pocket allowance (in Rs.) 11-13 13-15 15-17 17-19 19-21 21-23 23-25

Number of children 3 6 9 13 k 5 4

OR

The following frequency distribution shows the distance (in metres) thrown by 68

Material downloaded from myCBSEguide.com. 5 / 17


5
students in a Javelin throw competition.

Distance (in m) 0-10 10-20 20-30 30-40 40-50 50-60 60-70

Number of students 4 5 13 20 14 8 4

Draw a less than type Ogive for the given data and find the median distance thrown using
this curve.

Material downloaded from myCBSEguide.com. 6 / 17


6
CBSE Class-X Mathematics
Sample Paper (By CBSE)
Solution

1. Non terminating repeating decimal expansion


2.
3.
4.
5.
6. 25
7.

8. Sn = 2n2 + 3n

S1 = 5 = a1

S2 = a1 + a2 = 14 a2 = 9

d = a2 – a1 = 4

a16 = a1 + 15d = 5 + 15(4) = 65

9. For pair of equations kx + 1y = k2 and 1x + ky = 1


We have:

For infinitely many solutions,

From (i) and (ii), k = 1

10. Since is the mid-point of the line segment joining the points

therefore,
The line passes through the point
11. i. P(square number)
ii. P(multiple of 7)
12. Let number of red balls be = x

Material downloaded from myCBSEguide.com. 7 / 17


7
If 6 more red balls are added:
The number of red balls = x + 6

Since,
There are 3 red balls in the bag.
13. Let n = 3k, 3k + 1 or 3k + 2.
i. When n = 3k:
n is divisible by 3.
n + 2 = 3k + 2 n + 2 is not divisible by 3.
n + 4 = 3k + 4 = 3(k + 1) + 1 n + 4 is not divisible by 3.
ii. When n = 3k + 1:
n is not divisible by 3.
n + 2 = (3k + 1) + 2 = 3k + 3 = 3(k + 1) n + 2 is divisible by 3.
n + 4 = (3k + 1) + 4 = 3k + 5 = 3(k + 1) + 2 n + 4 is not divisible by 3.
iii. When n = 3k + 2:
n is not divisible by 3.
n + 2 = (3k + 2) + 2 = 3k + 4 = 3(k + 1) + 1 n + 2 is not divisible by 3.
n + 4 = (3k + 2) + 4 = 3k + 6 = 3(k + 2) n + 4 is divisible by 3.
Hence exactly one of the numbers n, n + 2 or n + 4 is divisible by 3.

14. Since are the two zeroes therefore,

is a factor of given polynomial.

We divide the given polynomial by

For other zeroes,

Material downloaded from myCBSEguide.com. 8 / 17


8
Zeroes of the given polynomial are

15. Let the ten’s and the units digit be y and x respectively.
So, the number is 10y + x.
The number when digits are reversed is 10x + y.
Now, 7(10y + x) = 4(10x + y) 2y = x …(i)
Also x – y = 3 …(ii)
Solving (1) and (2), we get y = 3 and x = 6.
Hence the number is 36
16. Let x-axis divides the line segment joining at the point P in the
ratio 1 : k.

Now, coordinates of point of division

Since P lies on x-axis, therefore

Hence the ratio is

Now, the coordinates of P are

OR

Let the height of parallelogram taking AB as base be h.

Now AB

17.

Since,

Material downloaded from myCBSEguide.com. 9 / 17


9
Also
And

OR

Construction: Draw

18. Join OC
In
OP = OC (radii of same circle)
PA = CA (length of two tangents)

AO = AO (Common)
(By SSS congruency criterion)
Hence,
Similarly

Material downloaded from myCBSEguide.com. 10 / 17


10
Now,

19. .

OR

20. We know, AC = r

Required area = + ar(semicircle on BC as diameter) –ar(quadrant ABPC

21. Let the area that can be irrigated in 30 minute be A


Water flowing in canal in 30 minutes
Volume of water flowing out in 30 minutes
Volume of water required to irrigate the field
Equating (i) and (ii), we get

Material downloaded from myCBSEguide.com. 11 / 17


11
OR

Surface area of remaining solid where r and l are the radius and
slant height of the cone.

22.

So, the mode marks is 68.


Empirical relationship between the three measures of central tendencies is:

23. Let original speed of the train be x km/h.


Time taken at original speed
Time taken at increased speed
Now,

OR

Discriminant
So, the given equation has two distinct real roots

Material downloaded from myCBSEguide.com. 12 / 17


12
Multiplying both sides by 5.

Verification:

Similarly,

24. Let the three middle most terms of the AP be a – d, a, a + d.


We have, (a – d) + a + (a + d) = 225

Now, the AP is
a – 18d,…,a – 2d, a – d, a, a + d, a + 2d,…, a + 18d
Sum of last three terms:

Now, first term


The AP is 3, 7, 11, …, 147.
25. Given: A right triangle ABC right angled at B.
To prove:
Construction: Draw
Proof: In

Now, (corresponding sides are proportional)

Material downloaded from myCBSEguide.com. 13 / 17


13
Similarly

Adding (1) and (2)

OR

Given:

To prove:

Construction: Draw

In


Therefore,

But

Hence,

26. Draw in which and hence


Construction of similar triangle as shown below:

Material downloaded from myCBSEguide.com. 14 / 17


14
27.

28.

Now,

Now,
Height of tower = TR = 75 m
Distance between building and tower

Material downloaded from myCBSEguide.com. 15 / 17


15
29. Capacity of mug (actual quantity of milk)

Amount dairy owner B should charge for one mug of milk


Value exhibited by dairy owner B: honesty (or any similar value)
30.

Daily pocket Number of Mid-point


allowance (in Rs.) children

11 - 13 3 12 -3 -9

13 – 15 6 14 -2 -12

15 - 17 9 16 -1 -9

17 – 19 13 18 0 0

19 – 21 K 20 1 k

21 – 23 5 22 2 10

23 - 25 4 24 3 12

OR

Less than Number of Students

10 4

20 9

30 22

40 42

Material downloaded from myCBSEguide.com. 16 / 17


16
50 56

60 64

70 68

Median distance is value of x that corresponds to Cumulative frequency


Therefore, Median distance = 36 m

Material downloaded from myCBSEguide.com. 17 / 17


17
CBSE Class-X Mathematics
Sample Paper 01

Time allowed: 3 Hours Max. Marks: 80


General Instructions:

i. All questions are compulsory.


ii. The question paper consists of 30 questions divided into four sections A, B, C and D.
iii. Section A contains 6 questions of 1 mark each. Section B contains 6 questions of 2 marks
each. Section C contains 10 questions of 3 marks each. Section D contains 8 questions of 4
marks each.
iv. There is no overall choice. However, an internal choice has been provided in four
questions of 3 marks each and three questions of 4 marks each. You have to attempt only
one of the alternatives in all such questions.
v. Use of calculators is not permitted.

Section-A

1. Can two numbers have 18 as their HCF and 380 as their LCM? Give reason.
2. Find the root of the equation .
3. Determine whether 50cm, 80cm, 100cm can be the sides of a right triangle or not.
4. The length of the shadow of a man is equal to the height of man. The angle of elevation is
________.
5. If the perimeter & area of a circle are numerically equal, then find the radius of the circle.
6. If 3 coins are tossed simultaneously, then find the probability of getting at least 2 heads.

Section-B

7. Is 7× 6 × 5 × 4 × 3 × 2 × 1+ 5 a composite number? Justify your answer.

8. The 11th term of an A.P. exceeds its 4th term by 14. Find the common difference.
9. Find the relation between ‘x’ and ‘y’, if the points , (1,2) and (7,0) are collinear.
10. Two tangents making an angle of 120° with each other are drawn to a circle of radius 6
cm, find the length of each tangent.
11. Prove that

Material downloaded from myCBSEguide.com. 1 / 14


18
12. A cone of height 20 cm and radius of base 5 cm is made up of modelling clay. A child
reshapes it in the form of a sphere. Find the diameter of the sphere.

Section-C

13. Use Euclid’s division lemma to show that cube of any positive integer is either of the form
9q, 9q+1 or 9q + 8 for some integer ‘q’.

14. Obtain all other zeroes of x4 + 5x3 - 2x2 - 40x -48 if two of its zeroes are
15. Solve for ‘x’: , x ≠ 0.
16. How many terms of the series 54, 51, 48,…….be taken so that their sum is 513? Explain the
double answer.

Or

In an AP pth, qth and rth terms are respectively a, b and c. Prove that
p(b - c) + q(c - a) + r(a - b) = 0
17. The point A(3, y) is equidistant from the points P(6,5) and Q(0, -3). Find the value of y.

Or

If A (4, 6), B (3, -2) and C (5, 2) are the vertices of Δ ABC, then verify the fact that a median
of a triangle ABC divides it into two triangles of equal areas.
18. If prove that
19. An observer 1.5m tall is 28.5m away from a chimney. The angle of elevation of the top of
the chimney from her eyes is 45°. What is the height of the chimney?

Or

From the top of a 7m high building, the angle of elevation of the top of a cable tower is

60o and the angle of depression of the foot of the tower is 30o. Find the height of the
tower.
20. A boy is cycling such that the wheels of the cycle are making 140 revolutions per minute.
If the diameter of the wheel is 60 cm, calculate the speed in Km per hour in which the boy
is cycling.
21. The following table shows the gain in weight by 50 children in a year. Calculate modal
gain in weight.

Material downloaded from myCBSEguide.com. 2 / 14


19
Gain in weight (in kg) 1 - 3 3 - 5 5 - 7 7 - 9 9- 11 11-13

No. of children 4 6 10 18 7 5

Or

Compute the Median for the given data

Class –interval 100-110 110-120 120-130 130-140 140-150 150-160

Frequency 6 35 48 72 100 4

22. What is the probability that a leap year, selected at random will contain 53 Thursdays?

Section-D

23. Solve graphically the following equations 2x + 3y = 9; x – 2y =1. Shade the region bounded
by the two lines and the x axis.

Or

Check graphically whether the pair of equations x + y = 8 and x – 2y = 2 is consistent. If so,


solve them graphically. Also find the coordinates of the points where the two lines meet
the y-axis.
24. A thief away from a Police Station with a uniform speed 100m/minutes. After one minute
a Policeman runs behind the thief to catch him. He goes at a speed of 100m/minute in first
minute and increases the speed 10m/minute on each succeeding minute. After how many
minutes the Policeman catches the thief.
Now answer these questions:
i. Which mathematical concept is being used to solve the above problem?
ii. Which trait of personality of the policeman is showed?
25. In the adjoining figure, PQR, is a right triangle, right angled at Q. X and Y are the points

on PQ and QR such that PX : XQ = 1 : 2 and QY : YR = 2 : 1. Prove that 9(PY2 + XR2) = 13 PR2

Material downloaded from myCBSEguide.com. 3 / 14


20
Or

A quadrilateral ABCD is drawn to circumscribe a circle (fig-2). Prove that, AB + CD = AD +


BC.

26. Prove that the lengths of two tangents drawn from an external point to a circle are equal.
27. Construct an isosceles triangle whose base is 7cm and altitude 5 cm and then construct
another triangle whose sides are times the corresponding sides of the isosceles
triangle.
28. Suppose a person is standing on a tower of height m and observing a car
coming towards the tower. He observed that angle of depression changes from 30° to 45°,
in 3 seconds. Find the speed of the car.
29. A container opens at the top and made up of metal sheet is in the form of a frustum of a
cone of height 16cm with diameters of its lower and upper ends as 16cm and 40cm
respectively. Find the cost of metal sheet used to make the container, if it costs Rs.10 per

100cm2. ( Use =3.14)


30. The mean of the following frequency distribution is 47. Find the value of ‘p’.

Classes 0 - 20 20 - 40 40 - 60 60 - 80 80 - 100

Frequency 8 15 20 p 5

Or

Compute the mode for the following frequency distribution.

Size of items: 0-4 4-8 8-12 12-16 16-20 20-40 24-28 28-32 32-36 36-40

Frequency: 5 7 9 17 12 10 6 3 1 0

Material downloaded from myCBSEguide.com. 4 / 14


21
CBSE Class-X Mathematics
Sample Paper 01
Solution

1. No. Because HCF is always a factor of LCM but here 18 is not a factor of 380.

2. 16x2 -27x-10=0
(16x + 5) (x - 2)=0
x= , x = 2
3.
Clearly, the sum of the squares of the lengths of two sides is not equal to the square of the
length of the third side. Hence, given sides do not make a right triangle because it does
not satisfy the property of Pythagoras theorem.
4. 45°
5. Perimetre of the Circle = Area of the Circle

6. Number of possible outcomes = 8 ( HHH, HHT, HTH,HTT, THH, THT, TTH, TTT)
Number of favorable outcomes ( 2 head) = 4
So probability =
7. Yes, 5040 + 5 = 5045 It has more than two factors
8. Let the first term of AP is a and d is common difference then According to Question
a11 – a4 =14 ;

a + 10d – (a + 3d) = 14
a + 10d - a - 3d = 14
7d = 14
d=2
9. [ ]

x = 7 – 3y
10.

Material downloaded from myCBSEguide.com. 5 / 14


22
PT=PS ( length of tangents)
60°
ΔOTP tan 60°=
PT= cm
11. LHS =

= RHS
12. Volume of cone = Volume of sphere

r = 5 cm
13. a = bq + r ; b =3 ; r = 0, 1, 2

a3 =(3m)3

= 9 (3m3)
= 9 q

a3 =(3m+1)3

= 27m3 + 27 m2 + 9m +1
= 9 q + 1

a3 =(3m+2)3

= 27m3 + 54 m2 + 18 m +8
= 9 q + 8

Material downloaded from myCBSEguide.com. 6 / 14


23
14. Two zeroes are
Therefore

15.

2x2 +2ax +bx +ab = 0


2x( x+a ) + b ( x+a) = 0
(x+a) (2x+b) = 0
x = -a ,
16. a = 54 , d= -3 , sn = 513

Sn =

513 =

n = 18 , 19
Since d is negative. we get double answer because sum of 18 terms and 19 terms is zero,
as few terms are positive and few are negative.

Or

A + (p - 1)D = a ..... (i)


A + (q-1) D=b …. (ii)
A + (r-1) D=c ...... (iii)
(ii) - (iii)
b - c = (q-1) D –(r-1)D

...... (iv)
Similarly,

Material downloaded from myCBSEguide.com. 7 / 14


24
...... (v)
....... (vi)
Adding (iv), (v) and (vi)
p(b - c) + q(c - a) + r(a - b) = 0
17. PA =QA 17. The point A(3, y) is equidistant from the points P(6,5) and Q(0, -3). Find the
value of y.

18.

19.

Given the height of the observer be DE = 1.5 m


That is AB = 1.5 m
Let BC = h is height of the chimney
Hence AC = (h – 1.5) m

Material downloaded from myCBSEguide.com. 8 / 14


25
Given distance between the observer and the chimney is
AD = BE = 28.5 m
In right triangle DCA, θ = 45°
tan 45°=

∴ h = 28.5 +1.5 = 30 m
Thus the height of the chimney is 30 m.
20. Circumference of wheel = πd = 60 π cm
Distance covered in 1 revolution = km
Distance covered in 140 revolution = km
(Distance covered in 1 min)
Distance covered in 1 hr =
Speed of cycle = 15.84 km / hr
21. Modal class = 7 - 9
Mode-

=
22. There are 366 days in a leap year that contain 52 weeks and 2 more days. So, 52
Thursdays and 2 days.
These 2 days can be:
(Mon, Tue}, {Tue, Wed}, {Wed, Thu}, {Thu, Fri}, {Fri, Sat}, {Sat, Sun} and {Sun, Mon} (7
cases).
In order to have 53 Thursdays we should have either {Thu, Fri} or {Wed, Thu} case.
No. of sample spaces = 7.
No. of event that gives 53 Thursdays in a leap Year = 2.
Required Probability =
23.

Material downloaded from myCBSEguide.com. 9 / 14


26
Table for Eqn.1 and its line
Table for Eqn.2 and its line
Solution x =3 and y =1
Shaded area is required solution.
Or

x 0 4 8

y = 8 –x 8 4 0

Three solutions for equation (i)are given in the table :

Three solutions for equation (ii) are given in the table :

x 0 2 8

–1 0 3

Drawing Line AC
Drawing Line PR

Material downloaded from myCBSEguide.com. 10 / 14


27
Plotting points A(0, 8), B(4, 4) and C(8, 0) on graph paper the straight line AC is obtained as
graph of the equation
(i) Plotting points P(0, –1), Q(2, 0) and R(8, 3) on graph paper the straight line PR is
obtained as graph of the equation
(ii) From the graph, it is clear that a point M(6, 2) common to both the lines AC and PR.
So the pair of equations is consistent and the solutions of the equations are x = 6 and y =
2.
From the graph it is seen that the coordinates of the points where the lines AC and PR
meets the y-axis are (0, 8) and (0, –1) respectively.
24. Time taken by Thief before being caught = n+1
Distance travelled by Thief = 100 ( n+ 1)
100 ( n+1) =

n = 5 minutes
i) Arithmetic Progression
ii) Responsibility of their work ( duty) and honesty
25.

Material downloaded from myCBSEguide.com. 11 / 14


28
Or

Since, the lengths of tangents drawn from an external point to a circle are equal.

AP = AS … (i) BP = BQ … (ii)
CQ = CR … (iii) DR = DS … (iv)
Now, AB + CD
= AP + PB + CR + RD
= AS + BQ + CQ + DS
= (AS + DS) + (BQ + CQ)
= AD + BC
Hence proved.
26. Construction: Draw a circle with centre O. From a point P outside the circle, draw two
tangents P and R.

To Prove: PQ = PR
Proof: In Δ POQ and Δ POR

Material downloaded from myCBSEguide.com. 12 / 14


29
OQ = OR (radii)
PO = PO (common side)
∠PQO = ∠PRO (Right angle)
ΔPOQ Δ POR (By RHS Congruency rule)
Hence proved
27.

28.

ΔPQB tan 45° = , x= 15( ------------ (1)

ΔPQA tan 30° = x + y = 15 ( --------- (2)

From (1) and (2) y = 30 m


Since the car moving from A to B in 3 seconds
Speed = = 10 m / sec
29. R = 20 cm , r = 8 cm , h= 16 cm

Material downloaded from myCBSEguide.com. 13 / 14


30
l = = 20 cm
Total surface area = CSA of frustum + area of base
= πl(R+r) +
= 1959.36

Rate of metal sheet used = Rs.10 per 100 cm2


Cost of metal sheet used = 1959.36 × = Rs.195.94 (Approximately)
30.

CI fi xi fix i

0 - 20 8 10 80

20 - 40 15 30 450

40 - 60 20 50 1000

60 - 80 p 70 70p

80 - 100 5 90 450

48+p 1980+70p

Material downloaded from myCBSEguide.com. 14 / 14


31
CBSE Class-X Mathematics
Sample Paper 02

Time allowed: 3 Hours Max. Marks: 80


General Instructions:

i. All questions are compulsory.


ii. The question paper consists of 30 questions divided into four sections A, B, C and D.
iii. Section A contains 6 questions of 1 mark each. Section B contains 6 questions of 2 marks
each. Section C contains 10 questions of 3 marks each. Section D contains 8 questions of 4
marks each.
iv. There is no overall choice. However, an internal choice has been provided in four
questions of 3 marks each and three questions of 4 marks each. You have to attempt only
one of the alternatives in all such questions.
v. Use of calculators is not permitted.

SECTION - A

1. The sum and the product of zeroes of a quadratic polynomial p(x) are 7 and 10
respectively. Then find p(x).
2. Which term of the A.P 92, 88, 84, 80, . . . . is 0?
3. Find the ratio in which the Y axis divides the line segment joining the points (5, -6) and
(-1, -4).
4. If PT is a tangent drawn from a point P to a circle touching it at T, and O is the centre of
the circle, then OPT + POT is ____________
5. If two towers of heights h1 and h2 subtend angles of 60° and 30° respectively, at the mid-

point of the line joining their feet, then h1 : h2 is ___________

6. In rolling a dice, the probability of getting a number less than 4 is ____________

SECTION - B

7. Solve for ‘x’ by completing the square method:

8. Find the 7th term from the end of the A.P. 7, 10, 13, . . . 184.

Material downloaded from myCBSEguide.com. 1 / 14


32
9. Find ‘a’ so that (3, a) lies on the line represented by 2x - 3y - 5 = 0. Also, find the
coordinates of the point where the line cuts the x axis.
10. ABC is a right triangle right-angled at B. Let D and E be any points on AB and BC

respectively. Prove that AE2 + CD2 = AC2 + DE2.


11. Two concentric circles have radii 5 cm and 3 cm. Find the length of the chord of the
larger circle which touches the smaller circle.
12. A cylinder and a cone have base radii 5 cm and 3 cm, respectively, and their respective
heights are 4 cm and 8 cm. Find the ratio of their volumes.

SECTION - C

13. A part of monthly expenses of a family is constant and the remaining varies with the
price of rice. When the cost of rice is Rs.250 per quintal, the monthly expenditure of the
family is Rs.1000 and when the cost of rice is Rs.240 per quintal the monthly expenditure
is Rs.980. Find the monthly expenditure of the family when the cost of rice is Rs.300 per
quintal.
14. Prove that is irrational.
15. Use Euclid’s division algorithm to find the HCF of 4052 and 12576.
16. The Sum Sn of first ‘n’ even natural numbers is given by the relation Sn= n(n+1). Find n, if

the sum is 420.

Or

A man arranges to pay a debt of Rs.3600 in 40 monthly installments which are in a AP.
When 30 installments are paid he dies leaving one third of the debt unpaid. Find the
value of the first installment.
17. Find the coordinates of the points of trisection of the line segment joining the points A( 1,
-2) and B(-3, 4).

Or

If the points (p, q) (M, n) and (p – m, q –n) are collinear, show that pn = qm.

18. In an equilateral triangle ABC, if AD BC, then prove that 3AB2= 4AD2.
19. In the adjoining figure, two tangents PQ and PR are drawn to a circle with Centre O from
an external point P. Prove that

Material downloaded from myCBSEguide.com. 2 / 14


33
Or

A sector of circle of radius 12 cm has the angle 120⁰ . It is rolled up so that the two
bounding radii are formed together to form a cone. Find the volume of the cone.
20. Find the value of
21. Two different dice are thrown together. Write all the possible outcomes. Find the
probability that the product of the numbers appearing on the top of the dice is less than
9.

Or

Anil bags contain 5 red marbles, 8 white marbles and 4 green marbles. One marble is
taken out of the box at random. What is the probability that the marble taken out will be
i. Red
ii. White
iii. Not green
22. The king, queen and jack of clubs are removed from a deck of 52 playing cards. The
remaining cards are well shuffled and one card is drawn at random from it. Find the
probability of getting the selected card as (i) a heart (ii). a king (iii). a club

SECTION - D

23. A two-digit number is such that the product of its digits is 12. When 36 is added to this
number, the digits interchange their places. Find the number.

Or

Solve for x, y:

24. A contract on construction job specifies a penalty for delay of completion beyond a

Material downloaded from myCBSEguide.com. 3 / 14


34
certain date as follows: Rs. 200 for the first day, Rs. 250 for the second day, Rs.300 for the
third day, etc., the penalty for each succeeding day being Rs.50 more that for preceding
day.
i. How much money the contractor has to pay as penalty, if he has delayed the work by
30 days?
ii. Which moral value one can learn from above problem?
25. Draw a pair of tangents to a circle of radius 4 cm, which are inclined to each other at an
angle of 60°.

Or

Construct a tangent to a circle of radius 2 cm from a point on the concentric circle of


radius 2.6 cm and measure its length. Also verify the measurements by actual
caclulations.(length of tangent = 2.1 cm)
26. Prove that
27. In the adjoining figure four equal circles are described at the four corners of a square so
that each touches two of the others. The shaded area enclosed between the circles is

cm2. Find the radius of each circle.

28. The angle of elevation of an aeroplane from a point A on the ground is 600. After a flight

of 30 seconds, the angle of elevation changes to 300. If the plane is flying at a constant
height of m, find the speed in km/hr of the plane.
29. A right triangle having sides 15cm and 20cm is made to revolve about its hypotenuse.
Find the volume and Surface Area of the double cone so formed. (Use p =3.14)
30. During the medical check-up of 35 students of a class, their weights were recorded as
follows

Weight

Material downloaded from myCBSEguide.com. 4 / 14


35
(in kg) Less Less Less Less Less Less Less Less
than 38 than 40 than 42 than 44 than 46 than 48 than 50 than 52

No. of
0 3 5 9 14 28 32 35
Students

Draw a less than type ogive for the given data. Hence, obtain the median weight from the
graph and verify the result by using the formula.

Or

Compute the Mean and Median for the given data.

Class –interval 100-110 110-120 120-130 130-140 140-150 150-160

Frequency 6 35 48 72 100 4

Material downloaded from myCBSEguide.com. 5 / 14


36
CBSE Class-X Mathematics
Sample Paper 02
Solution

1. x2- 7x+10

2. 24th term is 0.
3. 5 : 1
4. 90°
5. 3 : 1

6.

7.

8. d = -3 ; a = 184
a7=a + 6d

= 184 + 6 (-3)
= 184 – 18
= 166
9. Since (3, a) lies on 2x – 3y -5 = 0,
2(3) – 3a – 5 =0
a = 1/3
Let the co-ordinate of the point which cuts the x axis be (x, 0).
2x – 3(0)-5 = 0.
x = 5/2
(5/2, 0) is the point of intersection with x – axis
10.

Material downloaded from myCBSEguide.com. 6 / 14


37
Since ABE is right triangle, right-angled at B

AE2 = AB2 + BE2 ...(i)


Again, DBC is right triangle right- angled at B

CD2 = BD2 + BC2 ...(ii)


Adding (i) and (ii) we get

AE2 + CD2 = (AB2 + BE2 )+( BD2 + BC2 )

AE2 + CD2 = (AB2 + BC2 )+( BE2 + BD2 )


Using Pythagoras theorem in ABC and DBE we have

AE2 + CD2 = AC2+DE2


11.

Here PQ is tangent to smaller circle.


OR PQ
Now in PRO,

PR2+OR2=OP2

PR2+32=52
PR=
PR = QR (Perpendicular from the centre of the circle bisects the chord)
Length of the chord of the larger circle = PQ = 2 PR = 2 × 4 = 8 cm.
12. Volume of cylinder : Volume of cone

= 25 : 6
13. Let the constant expenditure be Rs.x
Let the consumption of rice by the family be y quintals.
x + 250 y = 1000 ----------(1)
x + 240 y = 980 ----------(2)
Solving (1) and (2)

Material downloaded from myCBSEguide.com. 7 / 14


38
x = 500; y = 2
Total expenditure when the cost of rice is Rs.300 = x +300y
= Rs.1100
14. Suppose is rational

But irrational while is rational and an irrational number can never be equal to
a rational number.
Thus our assumption is wrong and hence is irrational
15. Since 12576 > 4052, we apply the division lemma to 12576 and 4052, to get
12576 = 4052 × 3 + 420
4052 = 420 × 9 + 272
420 = 272 × 1 + 148
272 = 148 × 1 + 124
148 = 124 × 1 + 24
124 = 24 × 5 + 4
24 = 4 × 6 + 0
The remainder has now become zero, so our procedure stops.
HCF of 12576 and 4052 is 4.
16. n (n+1)=420

n2 + n – 420 = 0
n = -21 , 20
n cannot be negative ∴ n=20

17.

Let P(x1,y1) , Q(x2, y2) divide AB into 3 equal parts.

P divided AB in the ratio of 1 : 2

P(x1, y1) =

Since Q is the midpoint of PB,

Material downloaded from myCBSEguide.com. 8 / 14


39
Q(x2, y2) = =

18.

Given: ABC is equilateral and AD BC.

To Prove: 3AB2 = 4 AD2


Proof:
In ABD, By Pythagoras theorem,

AB2 = AD2 + BD2


BD =

AB2 =

AB2

19. Given: - (i) PQ and PR are the tangents of the circle


(ii) O is the centre of the circle

To prove:
Construction : Join OR and OP
Proof : - Now OR PR (radius from point of contact is to tangent)
Similarly, OQ PQ
In quadrilateral OPQR
(angles in same segment) ….. (1)

Material downloaded from myCBSEguide.com. 9 / 14


40
Also
[from (1)]
………. (2)
From (1) and (2)

Hence proved
20.

=
=
= 1 + 1 – 2
= 0
21. S= { (1,1) , (1,2), (1,3), (1,4), (1,5), (1,6),
(2,1) , (2,2) , (2,3) , (2,4), (2,5) , (2,6),
(3,1) , (3,2) , (3,3) , (3,4), (3,5) , (3,6),
(4,1) , (4,2) , (4,3) , (4,4), (4,5) , (4,6),
(5,1) , (5,2) , (5,3) , (5,4), (5,5) , (5,6),
(6,1) , (6,2) , (6,3) , (6,4), (6,5) , (6,6)}
n(S) = 36
Favourable events:(1,1) , (1,2) , (1,3) , (1,4), (1,5) , (1,6), ( 2,1) , (2,2) ,(2,3) , (2,4),(3,1) , (3,2) ,
(4,1) , (4,2) , (5,1), (6,1)
n(E) = 16
P(E) =
22. Total number of cards = 52
Number of cards removed = 3
n(s) = 52- 3 = 49
i. P(heart) =
ii. P(a king) =
iii. P(a club) =
23. Let Two digit number = 10x + y
xy=12 ------- (1)
10x + y + 36 = 10y + x
x – y = - 4 -------- (2)

from 1 and 2 we get x2 + 4x – 12 =0

Material downloaded from myCBSEguide.com. 10 / 14


41
x= -6 , 2
x cannot be negative , so x= 2
y=12/2 = 6
Required number is 26
24. The required sequence is 200, 250, 300, 350, ..........
a = 200, d = 50, n = 30

The contractor has to pay Rs. 27,750.


Value: One should be punctual and show dedication to his work, failing of which may
result loss.
25.

Draw a circle with radius 4 cm.


Draw 2 radius OQ and OR at 120°.
Construct s at Q, R to meet at P.
Join OQ and OR and they are required tangents.
26. LHS =

Material downloaded from myCBSEguide.com. 11 / 14


42
27.

Let r cm be the radius of each circle


Area of square - Area of 4 sectors =
Radius of each circle is 2 cm. (radius cannot be negative)

6r2 =24

r2 =4
r = ±2
28.

Let P be the position of the flight initially.


And BP be the height of the flight from the ground.
Consider ABP,

Material downloaded from myCBSEguide.com. 12 / 14


43
Consider ACQ,

BC = AC – AB=10,800 – 3600 = 7200 m


Speed of the flight =
= 864 km/hr
29. OA = 12 cm

Volume of double cone

= 3768 cm3
Surface area of double cone

= 1318.8cm2
30. By locating upper limits on X axis and number of students on Y axis, we get the following

Material downloaded from myCBSEguide.com. 13 / 14


44
graph

From the graph, we see that median is 46.5.


The frequency distribution table with the given cumulative frequencies becomes:

Class Intervals Frequency Cumulative Frequency

Below 38 0 0

38-40 3 3

40-42 2 5

42-44 4 9

44-46 5 14

46-48 14 28

48-50 4 32

50-52 3 35

Here n/2 = 35/2 = 17.5.


Now 46 – 48 is the class whose cumulative frequency is 28 is greater than n/2 i.e17.5.
So, 46 – 48 is the median class.
From the table f = 14, cf = 14, h =2

Median =

= 46 + 0.5
= 46. 5

Material downloaded from myCBSEguide.com. 14 / 14


45
CBSE Class X Mathematics
Sample Paper 03

Time allowed: 3 Hours Max. Marks: 80


General Instructions:

i. All questions are compulsory.


ii. The question paper consists of 30 questions divided into four sections A, B, C and D.
iii. Section A contains 6 questions of 1 mark each. Section B contains 6 questions of 2 marks
each. Section C contains 10 questions of 3 marks each. Section D contains 8 questions of 4
marks each.
iv. There is no overall choice. However, an internal choice has been provided in four
questions of 3 marks each and three questions of 4 marks each. You have to attempt only
one of the alternatives in all such questions.
v. Use of calculators is not permitted.

Section A

1. Find the value of so that the point , lies on the line represented by
2. Write the next term of the AP:
3. If the value of .
4. If areas of two similar triangles are in ratio 25: 64, write the ratio of their corresponding
sides.
5. In figure, CP and CQ are tangents to a circle with centre O . ARB is another tangent
touching the circle at R. If CP , BC = 7 cm, then find the length of BR .

6. A bag contains 4 red and 6 black balls. A ball is taken out of the bag at random. Find the
probability of getting a black ball.

Section B

7. Prove that is an irrational number.

Material downloaded from myCBSEguide.com. 1 / 20


46
8. Solve:

9. Find the value of p so that the quadratic equation has two equal
roots.

10. Evaluate :

11. The angle of elevation of the top of the tower from a point on the ground, which is 30 m
away from the foot of the tower, is 30 . Find the height of the tower.
12. A solid sphere of radius 10.5 cm is melted and recast into smaller solid cones, each of

radius 3.5 cm and height 3 cm. Find the number of cones so formed.

Section C

13. In a flight of 2800 km, an aircraft was slowed down due to bad weather. Its average speed
is reduced by 100 km/h and time increased by 30 minutes. Find the original duration of
the flight.

14. The sum of 4th and 8th term of an AP is 24 and sum of 6th and 10th terms is 44. Find AP.
15. If one diagonal of trapezium divides the other diagonal in the ratio 1: 3. Prove that one of
the parallel sides is three times the other.
16. Find the area of quadrilateral ABCD whose vertices are A(-4,-2), B(-3,-5), C(3,-2) and D(2,3).

Or

The points A(4, –2), B(7, 2), C(0, 9) and D(–3, 5) form a parallelogram. Find the length of
the altitude of the parallelogram on the base AB.
17. An aeroplane, when 3000 m high passes vertically above plane at any instant, when the
angle of elevation of the two aeroplanes from the same point on the ground are and
respectively. Find the vertical distance between aeroplanes.

Or

The angles of depression of the top and bottom of a building 50 metres high as observed
from the top of a tower are 30° and 60°, respectively. Find the height of the tower and
also the horizontal distance between the building and the tower.
18. If all sides of a parallelogram touch a circle, show that the parallelogram is rhombus.
19. Construct a triangle ABC in which BC = 8 cm, B = and C . Construct another

Material downloaded from myCBSEguide.com. 2 / 20


47
triangle similar to ABC such that its sides are of the corresponding sides of
ABC.

Or

Draw a triangle ABC with side BC = 7 cm, . Then, construct a


triangle whose sides are times the corresponding sides of ABC.
20. Show that the point A (3, 5), B (6, 0), C (1, -3) and D (-2, 2) are the vertices of a square
ABCD.
21. The median of distribution given below is 14.4. Find the values of and , if the sum of
frequency is 20.

Class interval 0-6 6-12 12-18 18-24 24-30

Frequency 4 x 5 y 1

Or

Find the mean, mode and median for the following data:

Classes 10-20 20-30 30-40 40-50 50-60 60-70 70-80

Frequency 4 8 10 12 10 4 2

22. Two dice are thrown simultaneously. What is the probability that
a. 5 will not come up on either of them?
b. 5 will come up on at least one?
c. 5 will come up on at both Dice?

Section D

23. Use Euclid’s Division Algorithm to show that the square of any positive integer is either of
the form or 3m+1 for some integer m.

Or

Show that exactly one of the numbers n, n + 2 or n + 4 is divisible by 3.


24. if the polynomial is divided by the
reminder comes out to be
25. Draw the graphs of the following equations: x + y = 5, x - y = 5
i. Find the solution of equation from the graph.

Material downloaded from myCBSEguide.com. 3 / 20


48
ii. Shade the triangular region formed by the lines and y-axis.

Or

Find the solution of pair of linear equation with the help of graph.2x + y = 6, 4x − 2y =
4
26. In an equilateral triangle ABC, D is a point on side BC such that 4BD = BC. Prove that

27. If prove that

28. In the given figure, three circle each of radius 3.5 cm are drawn in such a way that each
of them touches the other two. Find the area of shaded region enclosed between these
three circles. [Use

29. Water is flowing at the rate of 15 km/hour through a pipe of diameter 14 cm into a
cuboidial pond which is 50 m long and 44 m wide. In what time will the level of water in
pond rise by 21 cm?
30. Find the mean, mode and median of the following frequency distribution :

Class 0-10 10-20 20-30 30-40 40-50 50-60 60-70

Frequency 4 4 7 10 12 8 5

Or

The following distribution shows the daily pocket allowance of children of a locality. The
mean pocket allowance is Rs18. Find the missing frequency k.

Daily Pocket
11-13 13-15 15-17 17-19 19-21 21-23 23-25
Allowence

Number of
3 6 9 13 k 5 4
Children

Material downloaded from myCBSEguide.com. 4 / 20


49
CBSE Class X Mathematics
Sample Paper 03
Solution

1. Since point lies on line


Then

2.
AP is and so on
Common difference

Next term
3.

4. We know that,
If two triangle are similar then,

5. CP = CQ = 11 cm
[Tangents from same external point]
CQ = BC + BQ
But BQ = BR
11 = 7 + BR [as BC = 7 cm ]
BR = 4 cm
6. Number of ways t0o select a ball = 10
Number of ways to select a black ball = 6
Probability of getting a black ball =

Material downloaded from myCBSEguide.com. 5 / 20


50
7. Let, If possible, is a rational number.

From , we notice
LHS is an irrational number and RHS is rational number, which is not possible.
Hence, our supposition is wrong. Hence,
Is an irrational number.
8.

Adding (i) and (ii) we get

Subtract (i) from (ii) we get

Solving and we get


9.
Here,
For equal roots,D=0
D
0
0
But m [ In quadratic equation, q ]

10.


11. Let h be the height of the tower

Material downloaded from myCBSEguide.com. 6 / 20


51
In

12. Volume of solid sphere

Volume of 1 cone

Number of cones formed

13. Let original duration of flight be hours.


Distance km
Usual speed
When time is increased, then time taken hrs
And new speed

A.T.Q.,

Original duration of the flight hours

14. Let the 4th term

And 8th term


Then their sum

Similarly

Material downloaded from myCBSEguide.com. 7 / 20


52
On subtracting equation from we get

[From ]

Therefore, AP is
15.

DE = EB = 1 : 3
In (alt. angles)
(V.O.A.)

16.

Area of

Similarly, area of

Material downloaded from myCBSEguide.com. 8 / 20


53
Area of quadrilateral ABCD Area of Area of

Or

Let the height of parallelogram taking AB as base be h.


Now AB =
Area of
=
Now,

17. one aeroplane flying at point


Another aeroplane flying at point C. Both are in the same line.

In right

In right
From

Since

Material downloaded from myCBSEguide.com. 9 / 20


54
m

Vertical distance between two aeroplane = 3000-1732 m

Or

In

As BP = GR then From (i) and (ii)

Now, TR = TP + PR = (25 + 50) m.


Height of tower =TR = 75 m.
Distance between building and tower = GR =

Material downloaded from myCBSEguide.com. 10 / 20


55
18.

Sides of parallelogram ABCD, touch circle at P, Q, R, S


AP = AS, BP = BQ, CQ =CR, DR = DS
[Length of the tangents from a point outside the circle is equal ]
Consider, AB + CD AP + PB + CR + DR
AS + BQ +CQ +DS
(AS +SD ) + (BQ +QC)
AD + BC
AB + AB AD + AD
[ Opposite sides of a parallelogram are equal ]
2 AB 2AD
AB AD
AS Adjacent sides of a parallelogram are equal. Hence parallelogram is a rhombus
19.

Steps of construction:
1. Draw a line segment BC = 8 cm.
2. Then construct B = at B.
3. Then construct C = at C.
4. Line segment from the angles B and C, when produced, meet at A.
5. ABC is the constructed triangle.

Material downloaded from myCBSEguide.com. 11 / 20


56
6. Draw an acute angle CBX below BC.
7. Take points at BX, such that

8. Join
9. Draw parallel to meeting BC at C’.
10. Draw C’A’ parallel to CA, meeting BA at A’.

Or

Draw ABC in which BC = 7 cm, and hence C = 30°.


Construction of similar triangle A'BC' as shown below:

A’BC’ is the required triangle similar to ABC where each side is of the side of
ABC
20.

AB

BC

Material downloaded from myCBSEguide.com. 12 / 20


57
21.

Class interval Frequency cf

0-6 4 4

6-12 x 4+x

12-18 5 9+x

18-24 y 9+x+y

24-30 1 10+x+y

20

Median
Median class

Median

14.4

Or

We have

Classes Mid value (xi) fi fiu i c.f

10-20 15 4 -3 -12 4

20-30 25 8 -2 -16 12

30-40 35 10 -1 -10 22

40-50 45 12 0 0 34

50-60 55 10 1 10 44

60-70 65 4 2 8 48

70-80 75 2 3 6 50

Let assumed mean a = 45, Here h = 10 We know that mean

Material downloaded from myCBSEguide.com. 13 / 20


58
= 45 - 2.8
Mean
Since maximum frequency = 12
Modal class = 40-50
Here, l = 40, f1 = 12, f0 = 10, h = 10, f2 = 10

Now Mode

= 40 + 5
= 45
Mode = 45
Now
Median class is 40-50

Now median

Here N = 10, C = 22, F = 12, h = 10, l1 = 40

Median =

= 40 + 2.5
= 42.5
Thus, Median = 42.5
22. Elementary Events are

1 2 3 4 5 6

1 (1, 1) (1, 2) (1, 3) (1, 4) (1, 5) (1, 6)

2 (2, 1) (2, 2) (2, 3) (2, 4) (2, 5) (2, 6)

3 (3, 1) (3, 2) (3, 3) (3, 4) (3, 5) (3, 6)

4 (4, 1) (4, 2) (4, 3) (4, 4) (4, 5) (4, 6)

5 (5, 1) (5, 2) (5, 3) (5, 4) (5, 5) (5, 6)

6 (6, 1) (6, 2) (6, 3) (6, 4) (6, 5) (6, 6)

Total number of elementary events =36

Material downloaded from myCBSEguide.com. 14 / 20


59
a. Let A=5 will not come up on either of them.
Total number of elementary events favorable to A=25

b. Let B=5 will come up at least one die.


Total number of elementary events favorable to B=11

c. Let C=5 will come up at both dice.


Total number of elementary events favorable to C=1

23. Let n be any positive integer.

When


When N

(Where )
When


(Where
Square of any positive integer is either of the form
24. P ( )

Material downloaded from myCBSEguide.com. 15 / 20


60
( )

Equating the coefficients, we get

Also
Using
From and we get
25.

5 0

0 5

5 0

0 5

i. Both lines intersect at point (5,0) on x- axis


ii. Required portion is shaded in the graph.

Or

Material downloaded from myCBSEguide.com. 16 / 20


61
2x + y = 6, 4x − 2y = 4
For equation 2x + y – 6 = 0, we have following points which lie on the line

x 0 3

y 6 0

For equation 4x – 2y – 4 = 0, we have following points which lie on the line

x 0 1

y -2 0

We can clearly see that lines are intersecting at (2, 2) which is the solution.
Hence x = 2 and y = 2 and lines are consistent.
26. In equilateral
4BD=BC

Construction: Draw

In right AED,

Material downloaded from myCBSEguide.com. 17 / 20


62
In right

[using (i)]
27.

Now, LHS

28.

ABC is an equilateral triangle each of whose side is of length = 3.5 + 3.5 = 7 cm

ar( ABC) =

=
Area of 3 sectors
=
Area of shaded region

=
=
29. let the level of water rises in the tank in hours
Length of the water flow in hours = 15000 metres

Material downloaded from myCBSEguide.com. 18 / 20


63
Diameter of the pipe = 14 cm
Radius
Volume of water
Volume of water flow in hours in the pond

Volume of cuboidal pond with water level of height 21 cm

ATQ,

Hence, the level of water in the pond will rise by 21 cm in 2 hours.


30. Table for mean, mode and median :

C.I

0-10 5 4 -3 -12 4

10-20 15 4 -2 -8 8

20-30 25 7 -1 -7 15

30-40 35 10 0 0 25

40-50 45 12 1 12 37

50-60 55 8 2 16 45

60-70 65 5 3 15 50

Mean

For Median :
Median Class :

Median

For Mode: Maximum frequency


Modal Class is

Material downloaded from myCBSEguide.com. 19 / 20


64
Mode

Or

Daily Pocket Number of Mid-point


fiu i
Allowance Children(fi) (xi)

11-13 3 12 -3 -9

13-15 6 14 -2 -12

15-17 9 16 -1 -9

17-19 13 18 0 0

19-21 k 20 1 k

21-23 5 22 2 10

23-25 4 24 3 12

Material downloaded from myCBSEguide.com. 20 / 20


65
CBSE Class X Mathematics
Sample Paper 04

Time Allowed: 3 Hours Max. Marks: 80

General Instructions:

i. All questions are compulsory.


ii. The question paper consists of 30 questions divided into four sections A, B, C and D.
iii. Section A contains 6 questions of 1 mark each. Section B contains 6 questions of 2 marks
each. Section C contains 10 questions of 3 marks each. Section D contains 8 questions of 4
marks each.
iv. There is no overall choice. However, an internal choice has been provided in four
questions of 3 marks each and three questions of 4 marks each. You have to attempt only
one of the alternatives in all such questions.
v. Use of calculators is not permitted.

SECTION - A

1. Find the zeroes of the quadratic polynomial .


2. Write the sum of the first fifteen natural numbers.
3. Find the coordinate of the mid-point of the line segment joining the points whose

coordinates are and .

4. In fig-1, centre of the circle is O. From outside point P, two tangents PA and PB are drawn

to touch the circle. Given ÐAPB = 60o. Find ÐAOB.

5. A ladder of length 4 m. makes an angle of 45o with the ground when placed against an
electric post. Determine the distance between the feet of the ladder and the electric post.
6. Captain of Indian cricket team tosses two different coins, one of ₹1 and other of ₹2
simultaneously. What is the probability that he gets at least one head?

Material downloaded from myCBSEguide.com. 1 / 16


66
SECTION - B

7. Find the discriminant of the equation and hence find the nature
its roots.

8. If the 10th term of an A.P. is 47 and its 1st term is 2, find the sum of its first 15 terms.
9. Find a relation between x and y such that the point (x, y) is equidistant from the points (7,
1) and (3, 5).
10. In fig-2, and . Prove that, is isosceles.

11. Two concentric circles are of radius 5 cm and 3 cm. Find the length of the chord of the
larger circle which touches the smaller circle.
12. The height and base diameter of a solid cylinder are 210 m and 24 cm respectively. Find
the volume of the cylinder.

SECTION - C

13. The taxi charges in a city consist of a fixed charge together with the charge for the
distance covered. For a distance of 10 km, the charge paid is ₹ 200 and for a journey of 15
km, the charge paid is ₹275. What are the fixed charges and the charge per km? How
much a person has to pay for travelling a distance of 25 km?
14. Show that is irrational.
15. Show that any positive odd integer is of the form 4q + 1 or 4q + 3, where q is some integer.

16. The 4th term of an A.P. is equal to three times the 1st term & the 7th term exceeds twice

the 3rd term by 1. Find the 1st term and common difference.

Or

The first and the last terms of an AP are 17 and 350 respectively. If the common
difference is 9, how many terms are there and what is their sum?

Material downloaded from myCBSEguide.com. 2 / 16


67
17. If A(-5, 7), B(- 4, -5), C(-1, -6) and D(4, 5) are the vertices of a quadrilateral, find the area.

Or

Find the coordinates of the points which divide the line-segment joining the points (-4, 0)
and (0, 6) in four equal parts.
18. Prove that the sum of the squares of the sides of a rhombus is equal to sum of the squares
on its diagonals
19. Prove that, the lengths of tangents drawn from an external point to a circle are equal.

Or

In fig-3, PQ and RS are two parallel tangents to a circle with centre O and another tangent
EF with point of contact C intersecting PQ at E and RS at F. Prove that ∠EOF = 90°.

20. Evaluate:

21. A box contains 90 discs which are numbered from 1 to 90. If one disc is drawn at random
from the box, find the probability that it bears a perfect square number.

Or

Two horses are considered for a race. The probability of selection of the first horse is 1/4
and that of second is 1/3. What is the probability that :
a. both of them will be selected.
b. only one of them will be selected.
c. none of them will be selected.
22. A box contains 5 red marbles, 8 white marbles and 4 green marbles. One marble is taken
out of the box at random. What is the probability that the marble taken out will be
i) white ii) not green?

Material downloaded from myCBSEguide.com. 3 / 16


68
SECTION - D

23. Two water taps together can fill a tank in hours. The tap of larger diameter takes 10
hrs less than the smaller one to fill the tank separately. Find the time in which each tap
can separately fill the tank.

Or

A train, travelling at a uniform speed for 360 km, would have taken 48 minutes less to
travel the same distance if its speed were 5 km/hr more. Find the original speed of the
train.
24. Find the sum of all three-digit numbers which leaves the remainder 3 when divided by 5.
25. Construct a triangle similar to triangle ABC with its side equal to of the corresponding
sides of the triangle ABC.

Or

Construct an isosceles triangle whose base is 7cm and altitude 5 cm and then construct
another triangle whose sides are times the corresponding sides of the isosceles
triangle.

26. Prove that: .

27. In fig-3, of ABC is a right angle and AB = 3 units, AC = 4 units. Semicircles are
drawn on AB, AC and BC as diameters. Also a circle circumscribing the ABC is drawn.
Find the area of the shaded region.

28. Two ships are sailing in the sea on either side of a light-house. The angles of depression of

two ships as observed from the top of the light-house are 60o and 45o respectively. If the

distance between the ships is meters, find the height of the light-house.

29. A person connects a pipe of internal diameter 20m from a canal into a empty cylindrical
tank in his field which is 10 m in diameter and 2 m deep. If the rate of flow of water

Material downloaded from myCBSEguide.com. 4 / 16


69
through the pipe is 6km/hr, then after how much time the flow of water should be
stopped to avoid over flow of water from the tank ? What value we have from the
problem ?
30. The median of the following frequency distribution is 35. Find the value of x.

Class Interval 0-10 10-20 20-30 30-40 40-50 50-60 60-70

Frequency 2 3 5 6 x 3 2

Or

Compute the mode for the following frequency distribution.

Size of items: 0-4 4-8 8-12 12-16 16-20 20-40 24-28 28-32 32-36 36-40

Frequency: 5 7 9 17 12 10 6 3 1 0

Material downloaded from myCBSEguide.com. 5 / 16


70
CBSE Class X Mathematics
Sample Paper 04
Solution

SECTION- A

1. We have

=(x+5)(x+2)
So, the zeroes of are - 5 and - 2 .

2. The sum of the first fifteen natural numbers =

3. Coordinates of the mid-point are

4.
5.

6. Possible outcomes are (H, H), (H, T), (T, H), (T, T) which are equally likely.
Outcomes favorable to the event are (H, H), (H, T), (T, H).Hence the required Probability =

7. Here, .
Discriminant

Hence the given quadratic equation has two equal real roots.
8. Let the common difference of the A.P be d.

Given, the first term (a) = 2 and the 10th term ( ) = 47

As, nth term ,


So,
i.e., 9d = 47 - 2
i.e.,
As, the sum of the first n-terms of an AP,

Material downloaded from myCBSEguide.com. 6 / 16


71
Therefore, the sum of the first 15-terms of the AP

9. Since the point P(x , y) is equidistant from the points A(7, 1) and B(3, 5)
So, PA = PB i.e.,
i.e.,
i.e.,
i.e.,
i.e., , This is the required relqtion.
10.

Hence, (corresponding angles) ..... (i)


............ (ii)

Therefore, PQ = PR [sides opposite to equal angles ]


Hence, is isosceles.
11. Let C is the centre of two concentric circles of radii 5cm and 3 cm.

Let AB, a chord of the bigger circle touches the smaller circle at M.
CM = 3 cm, CA = 5 cm
[radius through point of contact is to tangent ]
[by Pythagoras Theorem ]
i.e.,
Therefore, AB=2AM = 2*4 = 8 cm.
[Line segment, drawn from centre of a circle perpendicular to any chord, bisects the
chord ]
Hence, the length of the chord is 8 cm.
12. Let V, h and r denote the volume, height and base radius of the cylinder respectively.

Material downloaded from myCBSEguide.com. 7 / 16


72
Here, h=210 m,
Then,

Hence the volume of the cylinder is


13. Let the fixed charge = ₹ x and the charge per km = ₹ y
According to the first condition, x + 10y = 200 …(i)
According to the second condition, x + 15y = 275 …(ii)
Now subtracting (i) from (ii), we get,
5y = 75
i.e., y = 15
From (i), x+ 10*15 = 200 (putting value of y ]
i.e., x = 200 - 150
i.e., x = 50
Hence, the fixed charge = 50 and the charge per km = ₹ 15
Now for travelling a distance of 25 km,
the person has to pay = ₹ (50 + 25*15) = ₹ 425.
14. let us assume that, is rational.
i.e., let for two coprime numbers a and b ( a, b are integers, b 0),

i.e.,

Since, is rational so, is rational.

This is a contradiction, because is irrational.


This contradiction is only for our incorrect assumption,
that, is rational.
Hence, the conclusion is that, is irrational.
15. According to Euclid’s division lemma,
Given positive integers a and b,
there exist unique integers q and r satisfying a = bq + r, 0 ≤ r < b.
Let us start with taking a, where a is a positive odd integer.
We apply the division algorithm with a and b = 4.
Since, 0 ≤ r < 4, the possible remainders are 0, 1, 2 and 3.
i.e., a can be 4q or 4q + 1 or 4q + 2, or 4q + 3, where q is the quotient.
However, since a is odd, a cannot be 4q or 4q + 2 (since they are both divisible by 2)

Material downloaded from myCBSEguide.com. 8 / 16


73
Therefore, any odd integer is of the form 4q + 1 or 4q + 3.
16. Let, a, d and be the first term, common difference and the term of the AP.
As,
According to the first condition,
i.e., a + (4 - 1)d = 3a
i.e., 3d = 2a …. (i)

Also, according to the 2nd condition,


i.e., a + (7 - 1)d - [a + (3 - 1)d ] = 1
i.e., a + 6d - a - 2d = 1
i.e., 4d = 1
i.e.,
Now from (i), i.e.,
Hence the first term = and common difference =
17. By joining A to C, we get and
As, area of a triangle with vertices at points

is
… (i)
So, using formula (i)

So the area of is .
Again, using formula (i),

So, the area of quadrilateral ABCD =


18. Let ABCD is rhombus.
Its diagonals AC and BD intersect at point O.

Material downloaded from myCBSEguide.com. 9 / 16


74
To prove that,
Since diagonals of a rhombus bisects each other perpendicularly,
so, AO = OC = ; BO = OD=
and AC is perpendicular to BD at point O.
and are all right angled triangle.
Using Pythagoras Theorem,
From ,
From ,
From ,
From ,
Adding (i), (ii), (iii), (iv), we get,
[ since AO = CO and BO = DO]
i.e.,

Hence proved
19. Let from an external point P two tangents PQ and PR
are drawn to the circle with centre at O.
The tangents touch the circle at points Q and R.

We have to prove that, PQ = PR.


Let us draw the line segments OQ, OR and OP.
Now in DPOQ and DPOR,

[each = 90o as OQ and OR are radii


through points of contacts]
OQ = OR [radii of same circle]
OP = OP [ Common side]

Material downloaded from myCBSEguide.com. 10 / 16


75
Therefore, [RHS]
Hence, PQ = PR [corresponding part of congruent triangles]
20. We know,


21. One disc is drawn at random from the box means that all the discs are equally likely to be
drawn.
Let the event of drawing one disc bearing a perfect square number be E.
Given that total number of discs in the box = 90
Therefore, total number of all possible outcomes = 90
1, 4, 9, 16, 25, 36, 49, 64, 81 are perfect square numbers between 1 to 90. 1 Therefore,
outcomes favorable to the event E = 9
So,

22. One marble is taken out of the box at random means,


all the marbles are equally likely to be taken out. Therefore, the total number of possible
outcomes = 5 + 8 + 4 = 17
Let the event of taking out of one white marble be W
and also let the event of taking out of one green marble be G
Then number of outcomes favorable to the event W = 8
Therefore, [answer of (i)]

Again the number of outcomes favourable to the event G = 4

Material downloaded from myCBSEguide.com. 11 / 16


76
So,

Therefore, P( not green) = 1 - P(G) [answer of (ii) ]


23. Let the tap of smaller diameter can fill the tank separately in x hrs
Then the tap of bigger diameter can fill the tank separately in (x - 10) hrs.
According to the question,
two water taps together can fill a tank in hours i.e., in hrs.
Then, in 1 hr smaller diameter tap can fill part of the tank,
in 1 hr bigger diameter tap can fill part of the tank,
and 1 hr two taps together can fill part of the tank,
So,
i.e.,

i.e.,
i.e.,
i.e.,
i.e.,
Now, using the quadratic formula, we get,

Since , so, x = 25
Therefore, tap of smaller diameter can fill the tank separately in 25 hrs, and the tap of
bigger diameter can fill the tank separately in 15 hrs.
24. The first three-digit number which leaves remainder 3 when divided by 5 is 103.[as 103 =
5*20 + 3 ]
Last three-digit number which leaves remainder 3 when divided by 5 is 998. [ as 998 =
5*199 + 3 ]
Now the three-digit numbers which leaves remainder 3 when divided by 5 are 103, 108,
111, …, 998, which form an AP.
First term, a = 103, common difference, d = 108 - 103 = 5
Let, 998 be the term ( ) of the series.
As,
So,

Material downloaded from myCBSEguide.com. 12 / 16


77
i.e.,
i.e.,

Therefore, 998 is the 180th term of the series.


Now sum of the terms of the series
=
=99090
25. Steps of construction:

Draw any ray BX making an acute angle with BC on the side opposite to the vertex A.
Locate 4 (the greater of 3 and 4 in ) points on BX so that

Join B4C and draw a line through B3 (the 3rd point, 3 being smaller of 3 and 4 in )

parallel to B4C to intersect BC at C′.

Draw a line through C′ parallel to the line CA to intersect BA at A′.


Thus, A′BC′ is the required triangle

26.

Material downloaded from myCBSEguide.com. 13 / 16


78

Hence proved.
27. In ABC, , AB=3 units, AC = 4 units
So, by Pythagoras Theorem,

Area of the semi-circle on BC (as diameter)

i.e., area of BECB =

Area of the semi-circle on AC (as diameter)

i.e., area of ADCA =


Area of the semi-circle on AB (as diameter)

i.e., area of AFBA =

Also from the fig. (area of AHBA + area of AGCA )


= area of semi-circle BHGCB - area of ABC

= -

= -

Area of AFBHA + area of AGCDA


= area of AFBA + area of ADCA - (area of AHBA + area of AGCA )

= + - sq units.

Material downloaded from myCBSEguide.com. 14 / 16


79
Hence, area of the shaded part
i.e., area of area of BECB +area of AGCDA + area of AFBHA

= + + - sq. units

= + + - sq.units

28. Let h meters be the height of the light-house AB.


Also let two ships be at C and D (in fig)

By question, CD = meters.

Angle of depression of two ships as observed

from the top (A) of the light-house are 60o and 45o respectively.
In the fig. ,
Therefore,
(alternate angle)
Now in ,

Again, in ,

[by (i)]

Hence, the height of the light-house is meters


29. Radius of the cylindrical tank = , its depth is 2 m.
Internal radius of the pipe =

Material downloaded from myCBSEguide.com. 15 / 16


80
So volume of the tank =

Flow of water through the pipe = 3 km/hr

Volume of water will flow through the pipe per minute


=
Therefore, time taken to fill the tank

So, flow of water should be stopped after 1 hr 40 min to avoid over flow from the tank.
Value: Water is most important substance in the earth for lives ( human, animals, plants)
so we should use our water wisely and not waste it.
30.

Class Interval Frequency Cumulative Frequency

0-10 2 2

10-20 3 5

20-30 5 10

30-40 6 16

40-50 x 16+x

50-60 3 19+x

60-70 2 21+x

Given median = 35. This lies in the class 30-40


So, l = 30, f = 6, cf = the cumulative frequency of the class preceding 30-40 = 10

We know,

Here,

Hence value of x is 5.

Material downloaded from myCBSEguide.com. 16 / 16


81
CBSE Class X Mathematics
Sample Paper 05

Time Allowed: 3 Hours Max. Marks: 80


General Instructions:

i. All questions are compulsory.


ii. The question paper consists of 30 questions divided into four sections A, B, C and D.
iii. Section A contains 6 questions of 1 mark each. Section B contains 6 questions of 2 marks
each. Section C contains 10 questions of 3 marks each. Section D contains 8 questions of 4
marks each.
iv. There is no overall choice. However, an internal choice has been provided in four
questions of 3 marks each and three questions of 4 marks each. You have to attempt only
one of the alternatives in all such questions.
v. Use of calculators is not permitted.

SECTION – A

1. Which term of the progression 4, 9, 14, 19, … is 109 ?


2. Find the sum of the zeroes of the quadratic polynomial .
3. If and , then what is the value of cot a ?
4. A die is thrown once. What is the probability of getting a composite number?
5. Find the coordinates of a point A, where AB is the diameter of a circle whose centre is (2,
– 3) and B is (1, 4).
6. In fig-1, AQ, AR and BC are the tangents. If AQ = 10 cm, find the perimeter of DABC.

SECTION – B

7. Determine an A.P whose 3rd term is 16 and difference of 7th term and 5th term is 12.

Material downloaded from myCBSEguide.com. 1 / 15


82
8. Find the coordinates of the points of trisection of the line segment joining points (4, –1)
and B(–2, –3).
9. Find two consecutive positive integers, sum of whose squares is 365.
10. In Fig–2, and AB = 6 cm. Find the length of DC.

11. In fig-3, from an outside point P, PA is a tangent to a circle whose centre is at C. A is the
point of contact. If PC = 10 cm and PA= 8 cm. Find the diameter of the circle.

12. 2 cubes each of volume are joined end to end. Find the surface area of the
resulting cuboid.

SECTION – C

13. Find two consecutive odd positive integers, sum of whose squares is 970.
14. How many terms of the sequence 18, 16, 14, … should be taken so that their sum is zero?
15. In fig-4, DE || BH and EF || HC. Show that, DF || BC.

16. Show that the points (1, 7), (4, 2), (–1, –1) and (– 4, 4) are the vertices of a square.

Or

Find the area of the rhombus, if its vertices are (3,0), (4,5), (-1,4) and (-2,-1) taken in order.
17. The ages of workers in a factory are given below :

Age (in years) 20-30 30-40 40-50 50-60 60-70

Number of workers 5 26 78 104 98

Material downloaded from myCBSEguide.com. 2 / 15


83
Find the modal age of workers.

Or

Compute the Median for the given data.

Class –interval 100-110 110-120 120-130 130-140 140-150 150-160

Frequency 6 35 48 72 100 4

18. Two dice one blue and one grey, are thrown at the same time. What is the probability
that the sum of the two numbers appearing on the top of the dice is 6?
19. Show that is irrational.

Or

Use Euclid’s division algorithm to find the HCF of 4052 and 12576.
20. Prove that,
21. In fig–5 ABCD is a quadrant of a circle with radius 28 cm and a semicircle BEDB is drawn
with BD as diameter. Find the area of the shaded region.

Or

The radius of a radius of a circle is 20cm Three more concentric circles are drawn inside
it in such a manner that it is divided into four parts of equal area. Find the radius of the
largest of the three concentric circle
22. Find the HCF of 96 and 404 by the prime factorization method. Hence, find their LCM.

SECTION – D

23. Construct a tangent to a circle of radius 4 cm from a point on the concentric circle of
radius 6 cm.
24. Five years hence, the age of Anubhab will be three times that of his son. Five years ago,
Anubhab’s age was seven times that of his son. What are their present ages?

Material downloaded from myCBSEguide.com. 3 / 15


84
25. Prove that: .

Or

If a cosθ – b sin θ = c prove that


26. The king, queen and jack of clubs are removed from a deck of 52 playing cards and the
remaining cards are shuffled. A card is drawn from the remaining cards. Find the
probability of getting a card of (i) heart (ii) king (iii) club.
27. In fig-6, AB and CD are two diameters of a circle with center at O. . OD is the
diameter of the smaller circle.Given that, OA = 7 cm, find the area of the shaded region.

28. From the top of a building 100 m. high, the angles of depression of the top and bottom of

a tower are observed to be 45o and 60o respectively. Find the height of the tower. Also
find the distance between the foot of the building and the bottom of the tower.

29. A wooden article was made by scooping out a hemisphere from each end of a solid
cylinder, as shown in fig-7. If the height of the cylinder is 10 cm and its base is of radius

3.5 cm, find the total surface area of the article.

30. The sum of first six terms of an A.P is 42. The ratio of its 10th term to its 30th term is 1 : 3.

Find the 1st term and the 13th term of the A.P.

Or

For what values of n nth term of the series 3, 10, 17...And 63, 65, 67..Are equal.

Material downloaded from myCBSEguide.com. 4 / 15


85
CBSE Class X Mathematics
Sample Paper 05
Solution

SECTION – A

1. 4+(n – 1).5 = 109


(n – 1).5 = 105
n = 21 + 1=22
2.

are the two zeroes.


So, sum the zeroes =
3. Given =

4. From 1 to 6 there are two composite numbers 4 and 6.


P(getting a composite number)

5. Let (x, y) be the coordinates of point A.

Coordinates of A are (3, –10).


6. Perimeter of ABC =AB+BC+AC

= (AQ – QB)+(BS + SC)+(AR – RC)


=AQ – BS + BS + RC +AQ – RC = 2AQ = 20 cm.

Material downloaded from myCBSEguide.com. 5 / 15


86
[Since The lengths of tangents drawn from an external point to a circle are equal.

7. n-th term, , a = 1st term and d = common difference of the AP


Now d = 6
Also, a+ (3 – 1)d = 16 or, a + 12 = 16 or, a = 4.
Therefore the terms of the AP are 4, 10, 16, 22, 28, …
8. Let point P(x, y) trisects the line segment joining points (4, –1) and B(–2, –3) [P is nearer to
the first point (4, –1) ]
Then, coordinates of P are

Again let point Q(x, y) trisects the line segment joining points (4, –1) and B(–2, –3)
[Q is nearer to the second point (–2, –3)]
Then, coordinates of Q are

9. Let the consecutive positive numbers be x and (x+1)


According to the question,

x = 13, since x is positive,


Hence the required numbers are 13 and 14
10. We know that, if one angle of a triangle is equal to one angle of the other triangle and the
sides including these angles are proportional, then the two triangles are similar.

Here, [vertically opposite]

Material downloaded from myCBSEguide.com. 6 / 15


87
Given,
Therefore, ~ .
i.e.,
Hence, i.e., DC = 12 cm.
11. We know that, The tangent at any point of a circle is perpendicular to the radius through
the point of contact.

So, .
By Pythagoras theorem,

Hence the diameter of the circle = 2*6 cm = 12 cm.


12. Length of each side of the cube of volume is 4 cm. If 2 cubes each of volume
are joined end to end, then a cuboid will be formed. The dimensions of the cuboid
formed are (4 + 4) cm, 4 cm , 4 cm i.e., 8 cm, 4 cm and 4 cm. The surface area of the
resulting cuboid

13. Let the consecutive odd positive integers be (2x – 1) and (2x + 1). (x is positive integer
1]
According to question,

i.e.,

Hence the required numbers are (2 11 – 1) and (2 11 + 1) i.e., 21 and 23.


ALTERNATE PROCESS:
Let the consecutive odd positive integers be x and (x + 2) [ x 1 ]
According to question,

Material downloaded from myCBSEguide.com. 7 / 15


88
[since, x is odd positive integer]
x = 21.
Hence the required numbers are 21 and 23.
14. In the given sequence, first term, a = 18,common difference, d = 16 – 18 =14 – 16 = - 2 =
constant Therefore, the sequence is an AP.
Let the sum of the first n terms is zero. We know, .
i.e.,

Hence, the sum of 19 terms of the sequence is zero.


15. By question DE || BH.
Therefore in ABH,
[Since, any line, drawn parallel to one side of a triangle, divides the other two sides in the
same ratio.]
Also by question, EF || HC.

So, [by same reason ]


Therefore by (i) and (ii),

We know that, if a line divides any two sides of a triangle in the same ratio, then the line
is parallel to the third side.
DF || BC [ by (iii)]
Hence proved.
16. Let (1, 7), (4, 2), (–1, –1) and (– 4, 4) are the coordinates of the points A, B, C and D
respectively.

Material downloaded from myCBSEguide.com. 8 / 15


89
Length of ,

Length of

Length of

Length of

Now,
Since, the lengths of the sides are equal, so ABCD is either a square or a rhombus.

Length of

Length of

Again,
i.e., lengths of the diagonals are equal. So ABCD is a square.
Hence, the given points are the vertices of a square.

Or

Let (3, 0), (4, 5), (−1, 4) and (−2, −1) are the vertices A, B, C, D of a rhombus ABCD

Length of diagonal

Length of diagonal

Therefore, area of rhombus ABCD


= 24 square units
17. From the given data, we se that the maximum frequency is 104 and the corresponding
class is 50-60.
Therefore, the modal class is 50-60.
Here, l = 50, h = 10, f = 104, and

Material downloaded from myCBSEguide.com. 9 / 15


90
Therefore,

i.e.,

Hence the modal age of workers is 58.125 years.


18. When the blue die shows 1, the grey die can show any one of the numbers 1, 2, 3, 4, 5, 6.
The same is true when the blue die shows 2, 3, 4, 5, 6. So, the number of possible
outcomes = 6 × 6 = 36.
Let the event that, ‘the sum of the two numbers appearing on the top of the dice is 6’ be
denoted by E.
Thereforem the outcomes favourable to the event E are:
(1, 5), (2, 4), (3, 3), (4, 2), (5, 1)
So number of outcomes favourable to the event E = 5
Therefore,

19. If possible let us assume, that, is a rational number.


i.e., , where p and q are coprime numbers and
i.e.,
As, 5, p, q are all integers so is rational.
Now from (i) we get that is also a rational.
But this contradicts the fact that is always an irrational number.
Therefore our assumption is wrong.
Hence we conclude that, is irrational.
20.

.
Hence proved.
21. Since ABCD is a quadrant of a circle with radius 28 cm,

Material downloaded from myCBSEguide.com. 10 / 15


91
so, AB = AD = 28 cm.
Area of the quadrant ABCD = sq. cm
area(BDCB) = area(quadrant ABCD) – area ( ABD)
= sq. cm
By Pythagoras Theorem,

Now, area of the shaded region = area (semi-circle BEDB) – area(BDCB)


=
= sq. cm
22. By prime factorization, of 96 and 404 can be written as
and
Therefore, the HCF of 96 and 404 is
Since Product of two numbers = (their HCF) (their LCM)
Hence LCM of 96 and 404 =

23.

Material downloaded from myCBSEguide.com. 11 / 15


92
Steps of construction :
Draw a circle of 4 cm radius with centre as O.
Draw a circle of 6 cm radius taking O as its centre.
Locate a point P on this circle and join OP.
Bisect OP. Let M be the mid-point of OP.
Take M as its centre and MO as its radius, draw a circle. Let it intersect the given circle at
the points Q and R.
Join PQ and PR.
PQ and PR are the required tangents.
24. Let the present age of Anubhab and his son be x years and y years respectively. Five
years hence, age of Anubhab will be (x+5) years and five years hence his son’s age will be
(y+5) years. By question, (x+5) = 3(y+5)
i.e., x – 3y = 15– 5
i.e., x – 3y = 10 … (i)
Five years ago, Anubhab’s age was (x–5) years and that of his son was (y – 5) years.
By question, (x–5)=7(y – 5)
i.e., x – 7y = 5–35
i.e., x – 7y = –30 … (ii)
Now subtracting (ii) from (i),

Subtracting ...... (ii) y = 10


From (i), x= 10 + 3 10 = 40
Hence, the present ages of Anubhab and his son are 40 years and 10 years.
25.

[Multiplying numerator and denominator by (cosecA – cotA)]

[multiplying numerator and denominator by (1-cosA)]

[ Dividing numerator and denominator by sinA ]


Hence proved.

Material downloaded from myCBSEguide.com. 12 / 15


93
26. After removing king, queen and jack of clubs from a deck of 52 playing cards there are 49
cards left in the deck. Out of these well shuffled 49 cards one card can be chosen in 49
different ways. Therefore number of total outcomes = 49
i. There are 13 heart cards in the deck of well shuffled 49 cards out of which one heart
card can be chosen in 13 different ways. Therefore, number of outcomes favourable
to the event of getting heart = 13
P(getting one heart card ) =
ii. There are 3 king cards in the deck of well shuffled 49 cards out of which one king can
be chosen in 3 different ways.
P(getting one king ) =
iii. There are (13 – 3) i.e., 10 club cards in the deck of well shuffled 49 cards out of which
one club can be chosen in 10 different ways.
P(getting one club card ) =
27. According to question, OA= 7 cm.
Therefore, OA = OB = OD = 7 cm.

Area of the circle with OD as diameter

Area of the semi-circle with OA as radius


=
Area of ABC =
Hence the area of the shaded area
= Area (circle with OD as diameter) + area(semi-circle with OA as radius) – ar( ABC)

28. Let AB and CD are the building and the tower respectively.

Material downloaded from myCBSEguide.com. 13 / 15


94
By question, AB = 100 m

Also by question, angles of depression of the top and bottom of a tower are observed

from the top of the building to be 45o and 60o respectively.


In the figure, .
Let, the height of the tower CD be h meter.
is drawn. EB = CD = h meter and AE = (100 – h) m
In DAEC, (alternate angle)
Then, in DAEC, i.e., EC = AE = (100 – h) m
BD= EC = (100 – h) m
Now in DABD, (al)
then,
i.e.,
i.e.,
i.e.,

Hence the height of the tower = 42.27 m (approx.)


The distance between the foot of the building and the bottom of the tower = 100 – 42.27 =
57.73 m
29. Let r and h are the base radius and height of the cylinder respectively.
Given, h = 10 cm and r = 3.5 cm.

Then radius of each of the hemisphere = 3.5 cm.


We know that, curved surface are of a cylinder = sq. units

Material downloaded from myCBSEguide.com. 14 / 15


95
and surface area of the hemisphere = sq. units
Total surface area of the article = Curved surface area of the cylinder + 2 (surface area
of one hemisphere)
= sq. units
sq. units

30. We know that, if a and d are the first term and the common difference of an AP, then,
term, and sum of first n terms,
According to the question,
i.e.,
i.e.,
Also by question,

i.e.,

i.e.,
i.e.,
i.e.,
i.e.,
Now from (i), 7d = 14, i.e., d = 2
a = d = 2

Now 13th term of the AP = 2+ (13 – 1)2 = 26

Hence the first term = 2 and the 13th term = 26.

Material downloaded from myCBSEguide.com. 15 / 15


96
CBSE Class X Mathematics
Sample Paper 06

Time allowed: 3 Hours Max. Marks: 80


General Instructions:

i. All questions are compulsory.


ii. The question paper consists of 30 questions divided into four sections A, B, C and D.
iii. Section A contains 6 questions of 1 mark each. Section B contains 6 questions of 2 marks
each. Section C contains 10 questions of 3 marks each. Section D contains 8 questions of 4
marks each.
iv. There is no overall choice. However, an internal choice has been provided in four
questions of 3 marks each and three questions of 4 marks each. You have to attempt only
one of the alternatives in all such questions.
v. Use of calculators is not permitted.

SECTION - A

1. Find the coefficient of x0 in x2+ 3x + 2 = 0


2. What is the common difference of the A.P. in which a18-a14=32?

3. If the points A(x, 2), B(-3,-4) and C(7, -5) are colinear, then find the value of x.
4. If PA and PB are tangents from an outside point P such that PA = 15 cm and APB = 60°.
Find the length of chord AB.
5. Find the length of the shadow of a 20m tall pole, on the ground when the sun’s elevation
is 45° .
6. If the probability of winning a game is 0.995, then find the probability of losing a game.

SECTION - B

7. Solve: .
8. Find the number of terms in the following series:
-5 + (-8) + (-11) + ……… + (-230)
9. Find the area (in sq.units) of the triangle formed by the points A(a, 0), O(0, 0) and B(0, b).
10. In two similar triangles ABC and PQR, if their corresponding altitudes AD and PS are in

Material downloaded from myCBSEguide.com. 1 / 14


97
the ratio 4 : 9, find the ratio of the areas of triangles ABC and PQR.

11. In the adjoining figure, O is the centre of the circle. AP and AQ two tangents drawn to the
circle. B is a point on the tangent QA and PAB = 125°, Find POQ.
12. The dimensions of a metallic cuboid are: 100 cm, 80 cm, 64 cm. It is melted and recast into
a cube. Find the surface area of the cube.

SECTION - C

13. The two forces are acting on a body such that their maximum and minimum value of
resultant is 27 N and 13 N respectively. Find the values of forces.

14. Prove that is irrational.


15. A sweet seller has 420 Kaju burfis and 130 Badam barfis. She wants to stack them in such
a way that each stack has the same number, and they take up the least area of the tray.
What is the maximum number of burfis that can be placed in each stack for this purpose?
(Use Euclid division algorithm)
16. Find the sum of all multiples of 9 lying between 100 and 500.

Or

Check whether 301 is a term of the list of numbers 5, 11, 17, 23, . . .
17. Find a point on y axis which is equidistant from (2, 2) and (9, 9).

Or

Find a relation between x and y if the points (2, 1) , (x, y) and (7, 5) are collinear
18. The hypotenuse of a right triangle is 6 m more than the twice of the shortest side. If the
third side is 2 m less than the hypotenuse, then find the sides of the triangle.
19. Prove that the parallelogram circumscribing a circle is a rhombus.

Or

An umbrella has 8 ribs, which are equally spaced, a ssuming umbrella to be a flat circle of

Material downloaded from myCBSEguide.com. 2 / 14


98
radius 45 cm. Find the area between two c onsecutive ribs of the umbrella.
20. If 7 cosec A - 3cot A = 7, prove that 7cot A - 3cosec A = 3.
21. A card is drawn from a well- shuffled pack of 52 playing cards. What is the probability
that the card drawn is
a. either a red or a king
b. a black face card
c. a red queen card.

Or

At a car park there are 100 vehicles, 60 of which are cars, 30 are vans and the remainder
are lorries. If every vehicle is equally likely to leave, find the probability of:
a. van leaving first.
b. lorry leaving first.
c. car leaving second if either a lorry or van had left first
22. From a bag containing 5 red, 6 black and 7 yellow balls, a ball is drawn at random. Find
the probability that it is:
a. not a yellow ball
b. neither a black nor a red ball
c. either a black or a yellow ball

SECTION - D

23. The sum of the areas of two squares is 640 m2. If the difference in their perimeters be
64m find the sides of the two squares.

Or

In a class test, the sum of marks obtained by Puneet in Mathematics and Science is 28.
Had he got 3 more marks in Mathematics and 4 marks less in Science, the product of
marks obtained in the two subjects would have been 180? Find the marks obtained in two
subjects separately.
24. A girl walking uphill covers a distance of 20 m during the first minute, 18m during the
second minute and 16m during the third minute and so on. How much distance will she

cover in (i) 10th minute (ii) 10 minutes?

Material downloaded from myCBSEguide.com. 3 / 14


99
25. Draw a circle of radius 8 cm. From a point 12 cm away from its centre, construct the pair
of tangents to the circle and measure their lengths.

Or

Draw a line segment AB of length 8 cm. Taking A as centre, draw a circle of radius 4 cm.
and constant the pair of tangents of the circle from point B and measure their lengths.

26. If sec A+ tan A= p, then prove that sin A = (p2+1)/(p2-1)


27. In the adjoining figure, PQ=24 cm, PR=7 cm and O is the centre of the circle. Find the area
of the shaded region.

28. From the top of a hill, the angles of depression of two consecutive kilometre stones due
east are found to be 30° and 45°. Find the height of the hill.
29. 21 glass spheres each of radius 2 cm are packed in a cuboidal box of internal dimensions
16 cm x 8 cm x 8 cm and then the box is filled with water. Find the volume of water filled
in the box.
30. The following frequency distribution gives the monthly consumption of electricity of 68
consumers of a locality. Find the mean, median and mode of the data.

Monthly Consumption 65-85 85-105 105-125 125-145 145-165 165-185 185-205

Number of Consumers 4 5 13 20 14 8 4

Or

The median of the following dats is 525. Find the value of x and y if the total frequencry is
100.

0- 100- 200- 300- 400- 500- 600- 700- 800- 900-


Classes
100 200 300 400 500 600 700 800 900 1000

Number of
2 5 X 12 17 20 Y 9 7 4
Consumers

Material downloaded from myCBSEguide.com. 4 / 14


100
CBSE Class X Mathematics
Sample Paper 06
Solutions

Section A

1. 2
2. d=8
3. -63
4. AB=15cm
5. 20cm
6. 0.005

Section B

7.

8. a = -5 ; d = -3
an = -230

-230= -5 + (n – 1) (-3)
-225 = (n-1)(-3)
75 = n-1
n = 76
9. Area of ABC =
= - sq.units
= sq.units ( as area cannot be negative)
10. Since the areas of two similar triangles are in the ratio of the squares of corresponding
altitudes,

Material downloaded from myCBSEguide.com. 5 / 14


101
Thus, the ratio of areas of triangles ABC and PQR = 16 : 81
11. PAB+ PAQ =180° [Linear Pair of angles]

PAQ + 125o = 180o

PAQ = 180o - 125o

PAQ = 55o
In Quadrilateral APOQ,
PAQ+ APO+ AQO+ POQ = 360°
55°+90°+90°+ POQ=360° [Radius is perpendicular with tangent at their point of contact]
POQ=360°- 235°
POQ =125°.
12. Volume of cube = Volume of cuboid

(side)3 = Length × Breadth × Height

(side)3 = 100×80×64
Side =
= 80 cm.

Surface Area of the Cube = 6a2 = 6 × 80 × 80 = 38400 cm2

Section C

13. Let the two forces in Newton(N) are x and y respectively.


Then,
x + y = 27 ------(1) [ For maximum force]
x - y = 13 -------(2) [ For minimum force]
Solving (1) and (2)
x = 20 ; y = 7
Required forces are 20N & 7N

14. Suppose is rational.

p and 1 are coprime

But is irrational while is rational and an irrational can never be equal to a


rational number.

Material downloaded from myCBSEguide.com. 6 / 14


102
Thus our assumption is wrong and hence is irrational.
15. The number of stacks will be least if the number of burfis in each stack is equal to the
HCF of 420 and130.
Now, let us use Euclid’s algorithm to find their HCF.
420 = 130 × 3 + 30
130 = 30 × 4 + 10
30 = 10 × 3 + 0
So, the HCF of 420 and 130 is 10.
Hence, the sweet seller can make stacks of 10m burfis of each kind to cover the least area
of the tray.
16. a = 108 , an = l = 495 ; d = 9

an = a + (n – 1) d

495 = 108 + (n -1) 9


n = 44
S44 = (a + l)

= (108 + 495)
= 13,266

Or

We have : a2 – a1 = 11 – 5 = 6,

a3 – a2 = 17 – 11 = 6,

a4 – a3 = 23 – 17 = 6

As ak + 1 – ak is the same for k = 1, 2, 3, etc.,

the given list of numbers is an AP.


Now, a = 5 and d = 6.
Let 301 be a term, say, the nth term of the this AP. We know that
an = a + (n – 1) d

So, 301 = 5 + (n – 1) × 6
i.e., 301 = 6n – 1 So,
n =
But n should be a positive integer.

Material downloaded from myCBSEguide.com. 7 / 14


103
So, 301 is not a term of the given list of numbers.
17. PA = PB

Squaring on both the sides,

Point is (0,11)

Or

If the given points A(2,1), B(x,y) and C(7,5) are collinear, then the area formed by these
points will be 0.

This is the requried relation between x and y.


18. Let the shortest side be x meters in length.
Then, hypotenuse = (2x+16)m
And the third side = (2x+4)m
By using Pythagoras theorem, we have

(2x + 6)2 = x2 + (2x + 4)2


(x – 10)(x + 2) = 0
x = 10 or x = -2
19. Let ABCD be parallelogram circumscribing circle with centre 0.

Material downloaded from myCBSEguide.com. 8 / 14


104
AS = AP (Tangents from A)
BP = BQ (Tangents from B)
CQ = CR (Tangents from C)
DS = DR (Tangents from D)\
Now, AP + BP + CR + DR = AS + BQ + CQ + DS
(AP + BP) + (CR + DR) = (AS + DS) + (BQ + CQ)
AB + CD = AD + BC
2AB = 2 BC
AB = BC
AB = BC = CD = DA
ABCD is a rhombus
20. 7cosecA – 3 cotA = 7
7cosecA – 7 = 3 cotA
7(CosecA – 1) = 3 cotA
Multiplying by (cosecA + 1) both sides, we get
7(cosecA – 1) (cosecA + 1) = 3 cotA (cosecA + 1)

7(cosec2A-1) = 3 cotA (cosecA + 1)

7cot2A=3 cotA (cosecA + 1)


7cotA = 3(cosecA + 1)
7cotA – cosecA=3
21. n(S)= 52
a. Let A be the event of drawing either a red or a king card
n(A) = 28
P(A) =
b. Let B be the event of drawing a black face card.
n(B) = 6
P(B) =

Material downloaded from myCBSEguide.com. 9 / 14


105
c. Let C be the event of drawing a red queen card.
n(C) = 2
P(C) =
22. a. let A be the event of drawing a ball other than yellow in colour.
n(A) = 11
P(A) =
b. Let B be the event of drawing a ball which is yellow in colour.
n(B) = 7
P(B) =
c. Let C be the event of drawing a ball which is either black or yellow .
n(C) = 13
P(C) =

Section C

23. Let the areas of two squares be a12 , a22

a12+ a22 = 640 ........ (1)

4 a1-4a2 =64

a1 – a2 = 16

a1 = 16 +a2

Substitute the value of a1 in (i) we get

(16 +a2)2 + a22 = 640

a22 + 16a2 -192 = 0

= 8, -24
But length cannot be negative.
a2 = 8

a1 = 24

Material downloaded from myCBSEguide.com. 10 / 14


106
Therefore the sides are 24 & 8.
24. The required sequence is 20,18,16,14, ..........
a = 20, d = -2, n = 10

i. 10th minute
an =a + (n -1) d

a10 =20 + 9(-2)

= 2 m

She covers a distance of 2 m during 10th minute.


ii. 10 minutes

= 5(22)
= 110
After 10 minutes she will cover 110 m.
25.

Steps of Construction:
Step 1: Draw a circle with centre O and radius 8 cm.
Step 2: Take a point P outside the circle such that OP = 12 cm. Join OP.
Step 3: Draw a perpendicular bisector of line segment OP. It intersects OP at O’.
Step 4: Take O’ as centre and O’O as radius and draw a circle which intersects the
previous circle at points A and B.
Step 5: Join P to A and P to B.
PA & PB are the required tangents from point P to circle with centre O.
Length of PA = PB = 14.4 cm
26. Let p (sec A + tan A)

Material downloaded from myCBSEguide.com. 11 / 14


107
By componendo and dividend method.

sinA = LHS
27. PQ = 24 cm, PR = 7 cm
Since P is a right angle, By Pythagoras theorem, we have

QR2 = PQ2 + PR2

= 242 + 72
= 576 + 49
= 625
QR = 25
Since QR is a diameter of a circle, radius of the circle r = 25/2 cm.
Area of the shaded region = Area of semi circle – Area of right triangle PQR

28. Let h = height of the hill


C and D are position of two stones.

Material downloaded from myCBSEguide.com. 12 / 14


108
In CAB, we have

In DAB, we have

29. Given, Radius of 1 glass sphere. R = 2cm

Therefore, Volume of such 21 glass spheres

Given, Dimensions of the cuboidal box = 16 cm × 8 cm × 8 cm


Therefore, Volume of the cuboidal box = 16 × 8 × 8
Volume of the water added in cuboid box =
Volume of the cuboidal box – volume of the 21 glass sphere

= 320 cm3
30.

Monthly No. of
Cumulative Class
consumption (in consumers di=xi-135 ui fiui
frequency(cf) mark
units) (fi)

65-85 4 4 75 -60 -3 -12

85-105 5 9 95 -40 -2 -10

105-125 13(f0) 22 115 -20 -1 -13

125 - 145 20(f1) 42 135 0 0 0

145-165 14(f2) 56 155 20 1 14

165-185 8 64 175 40 2 16

Material downloaded from myCBSEguide.com. 13 / 14


109
185 - 205 4 68 195 60 3 12

TOTAL 68 7

From the table, n = 68, n/2 = 34, h = 20

Mean =

Thus, the mean monthly consumption of electricity is 137.05 units.


Now, 125 - 145 is the median class because its frequency 42 is just greater than 34.

Median =

= 125 + 12
= 137 units
Model class is 125 – 145 because it has highest frequency

Mode =

= 125 + 10.76
= 135.77

Material downloaded from myCBSEguide.com. 14 / 14


110
CBSE Class X Mathematics
Sample Paper 07

Time allowed: 3 Hours Max. Marks: 80


General Instructions:

i. All questions are compulsory.


ii. The question paper consists of 30 questions divided into four sections A, B, C and D.
iii. Section A contains 6 questions of 1 mark each. Section B contains 6 questions of 2 marks
each. Section C contains 10 questions of 3 marks each. Section D contains 8 questions of 4
marks each.
iv. There is no overall choice. However, an internal choice has been provided in four
questions of 3 marks each and three questions of 4 marks each. You have to attempt only
one of the alternatives in all such questions.
v. Use of calculators is not permitted.

Section A

1. Is x = 2. y = 3 a solution of linear equation ?

2. The nth term of an AP is 7-4n. Find the common difference


3. Find the value of .
4. A ladder is 10 meter long reaches a window 8 meter above the ground. Find the distance
of the foot of the ladder from the base of the wall.
5. What is the relation between the lengths of two tangents from an external point to a
circle?
6. One card is drawn from well shuffled pack of 52 cards. Find the probability that the card
will be ace.

Section B

7. Find the highest positive integer by which dividing the numbers 396, 436, 542 reminders
5, 11, and 15 respectively.
8. For what value of p, the following system of equation have a unique solution :

Material downloaded from myCBSEguide.com. 1 / 25


111
9. Find two numbers whose sum is and product is

10. If then evaluate


11. The shadow of tower standing on a plane ground is found to be 40 m longer when the
Sun’s altitude reduces to from . Find the height of the tower.
12. A hemispherical tank full of water id emptied by a pipe at the rate of litre per second,
How much time will take to empty the full tank if it is 3m in diameter.

Section C

13. A pole has to be erected at a point on this boundary of a circular part of diameter 13
meters in such a way that the difference of its distances from two diametrically opposite
fixed gates A and B on the boundary is 7 meters. Is it possible to do so? If yes, at what
distance from the two gates should the pole be erected.
14. Find the sum of number between 1 to 100 divisible by 6.
15. In the given figure, the line segment XY is parallel to side AC of and it divides the
triangle into two parts of equal areas. Find the ratio .

16. Find the area of triangle formed by joining the mid points of the sides of the triangle ABC
whose vertices are A(0,-1), B(2,1) and C(0,3). Find the ratio of this area and the area of
ABC.

Or

Find the cirumcentre of the triangle whose vertices are (-2, -3), (-1, 0), (7, -6).
17. From a point on a bridge across a river the angles of depression of the banks on opposite
sides of the river are and respectively. If the bridge is at a height of 4m from the
banks, find the width of the river.

Material downloaded from myCBSEguide.com. 2 / 25


112
Or

If the angle of elevation of a cloud from a point 'h' metres above a lake is α and the angle
of depression of its reflection in the lake is β, prove that the height of the cloud is

18. Two concentric circles are if radii 25 cm. and 7 cm. respectively. Find the length of the
chord of the bigger circle which touches the smaller circle.
19. Draw a line segment AB of length 8 cm. Taking A as centre, draw a circle of radius 4 cm.
and constant the pair of tangents of the circle from point B and measure their lengths.

Or

Draw a circle of radius 8 cm. From a point 12 cm away from its centre, construct the pair
of tangents to the circle and measure their lengths.
20. Find the co- ordinate of the circumcentre of the triangle whose vertices are (8,6), (8,-2)
and (2, -2). Also its circum radius.
21. The distribution below shows the pocket money during a days of 60 employees in an
office. Find the median pocket money of the employees.

Pocket Money (in Rupees) 5-10 10-15 15-20 20-25 25-30 30-35 35-40

Number of Employees 3 6 9 11 14 10 7

Or

A survey conducted on 20 households in a locality by a group of students resulted in the


following frequency table for the number of family members in a household. Find the
mode.

Family size 1-3 3-5 5-7 7-9 9-11

No. of families 7 8 2 4 1

22. A box contains 30 discs which are number 1 to 30. If one disc is drawn at random from
the box. Find the probability that it hears –
i. A two digit number
ii. A perfect square number

Section D

Material downloaded from myCBSEguide.com. 3 / 25


113
23. Prove that is an irrational number.

Or

Use Euclid’s division lemma to show that the cube of any positive integer is of the form
9m, 9m + 1 or 9m + 8.
24. Divide by and verify the division algorithm.
25. Coach of a cricket team buys one bat and 2 balls for Rs. 300. Later he buys another 2 bates
and 3 balls of the same kind for Rs. 525. Represent this situation algebraically and solve it
by graphical method. Also find out that how much money coach will pay for the purchase
of one bat and one ball.

Or

The sum of two numbers is 16 and the sum of their reciprocals is . Find the numbers.

26. BE and CF are medians of triangle ABC right angled at A. Prove that 4(BE2 + CF2) = 5BC2
27. Prove that

28. Two circular flower beds lie on two sides AB and CD of a square lawn ABCD of side 56 m.
If the centre of each circular flower bed is point of intersection O of the diagonals of
square lawn. Find the sum of the areas of the lawn and flower beds.
29. A tent is the form of a cylinder of diameter 4.2 m and height 4 m, surrounding by a cone
of equal base and height 2.8 m. find the capacity of the tent and the cost of canvas for
making the tent at 100 per sq. m.
30. The median of the following dats is 525. Find the value of x and y if the total frequencry is
100.

0- 100- 200- 300- 400- 500- 600- 700- 800- 900-


Classes
100 200 300 400 500 600 700 800 900 1000

Number of
2 5 X 12 17 20 Y 9 7 4
Consumers

Or

Material downloaded from myCBSEguide.com. 4 / 25


114
The distribution below gives the marks of 100 students of a class.

Marks 0-5 2-10 10-15 15-20 20-25 25-30 30-35 35-40

No. of Students 4 6 10 10 25 22 18 5

Draw a less than type and a more than type ogive from the given data. Hence, obtain the
median marks from the graph

Material downloaded from myCBSEguide.com. 5 / 25


115
CBSE Class X Mathematics
Sample Paper 07
Solutions

1. The given equation is


Putting in given equation, we have

=
=
=
Hence, is a solution of given equation.
2. Here, an=7-4n

If n=1 then a1=7-4x1=3

If n=2 then a2=7-4x2=-1

Common difference d= a2- a1=-1-3=-4

3. .(putting the value of trigonometric ratios)

4. According to the figure ABC is a right angled triangle in which


Now, By the Pythagoras Theorem

5. Both tangent lines are equal.


6. There are 4 aces in a pack

Material downloaded from myCBSEguide.com. 6 / 25


116
The number of Favorable outcomes=4
The total number of outcomes=52
P (card is ace)
7. It is given that on dividing 396 by the required number, there is remainder of 5; this
means that 396-5 = 391 is exactly divisible by the required number.
Similarly 436 -11 = 425 is also exactly divisible by the required number and
542 – 15 = 527 is also exactly divisible by required number.
Also the required number is the largest number satisfying the above property.
Therefore it is HCF of 391, 425 and 527.
Let us now find the HCF of 391, 491 and 527
391 = 17 23
425 = 5 5 17
527 = 17 31
HCF (391, 425, 527) = 17
8. For unique solution

Or

9. Let one number be x


Anotthe number will be (27-x ) [ Sum of two number = 27]
Their product
According to the question

So the required two numbers are 13 and 14 Ans.


10.

Material downloaded from myCBSEguide.com. 7 / 25


117
11. Let AB be height of the tower and BC m

DB m
In ABC, we have

In ABD, we have

Substituting obtained from equation in equation

We get

Material downloaded from myCBSEguide.com. 8 / 25


118
12. radius of hemispherical tank m
Volume of the tank

So, the volume of water to be emptied litres


Litres
Since, litres of water is emptied in 1 second
litres of water will be emptied in second second

Or

minutes
Minutes.
13.

Let P the required location of the pole from the gate B be m, i.e. BP m. Now the
difference of the distance of the pole from the two gates AP-BP (or BP-AP) 7 m.
Therefore, AP = m.
Now, AB 13 m, an since AB is a diameter

(By Pythagoras theorem)
i.e.
i.e.
i.e.
So, the distance of the pole from gate B satisfies the equation

So, it would be possible to place the pole if this equation has real roots. To see if this is so
or not, let us consider it’s discriminant. This discriminant is

Material downloaded from myCBSEguide.com. 9 / 25


119
So, the given quadratic equation has two real roots, and it is possible to erect the pole on
the boundary of the park.
Solving the quadratic equation by the quadratic formula, we get

Therefore, or
Since is the distance between the pole and the gate B, it must be positive. Therefore,
will have to be ignored. So .
Thus the pole has to be erected on the boundary of the park at a distance of 5 m from the
gate B and 12 m from the gate A
14. The numbers between 1 to 100 divisible by 6

,
96 ,
Or ,
Or
Or
Or
Or
Or

15.

Material downloaded from myCBSEguide.com. 10 / 25


120
We have XY AC (Given)
So, BXY A and (Corresponding angles)
Therefore (AA Similarity Criterion)

So, (Theorem 6.6)

Also (Given)

So,

Therefore, from and

i.e. ,

or

or

or

or

16.

Let the vertices of ABC is A(0,-1), B(2,1) and C(0,3)


D,E,F are mid of AB, BC & CA.

Midpoint formula

Co-ordinate of D

Co-ordinate of E

Co-ordinate of F

Vertices of DEF is D(1,0) , E(1,2) , F(0,1)


Here

Material downloaded from myCBSEguide.com. 11 / 25


121
Area of DEF

In ABC

Ratio

Or

Let the centre of the circumcircle be O(x, y). The points are A(-2, -3), B(-1, 0) and C(7, -6).

OA = OB = Radius

Also, OB = OC = Radius

adding (i) and (ii), we get


5x = 15
x = 3

Material downloaded from myCBSEguide.com. 12 / 25


122
put value of x in (i) we get,
3y = -9
y = -3
So the circumcenter is (3,-3)
17.

In fig. A and B represent points on the bank on opposite sides of the river, so that AB is
the width of the river. P is a point on the bridge at a height of 4m , i.e. DP = 4m. we are
interested to determine the width of the river, which is the length of the side AB of the
APB.
Now, AB = AD + DB
In right APD,
So,

Also in right
So,

DB = 4m
AB = AD + DB
m
m
Therefore, the width of the river is m

Or

Let AN be the surface of the lake and O be the point of observation such that OA = h
meters

Material downloaded from myCBSEguide.com. 13 / 25


123
Let P be the position of the cloud and P' be its reflection in the lake.
Then PN = P'N
Let

From (i) and (ii), we get

Hence, height of the cloud is given by Pn = x + h

Material downloaded from myCBSEguide.com. 14 / 25


124
Hence proved.
18.

Let O be the centre of the concentric circles of radii 25 cm. and 7 cm. respectively. Let AB
be a chord of the larger circle touching the smaller circle at P. Then
AP = PB and OP AB
Applying Pythagoras theorem in OPA, We have

cm.
cm.
19.

Steps of Construction:
1. Draw a line segment AB = 8 cm.
2. Taking A as centre draw a circle of radius = 4cm.
3. Draw a perpendicular bisect of AB and let it intersect AB in P.
4. With P as centre and PA or PB as radius, draw another circle intersecting the given
circle in Q & R.
5. Join BQ and BR.
Thus BQ and BR are the required tangents from point P to the circle. Lengths BQ =BR =

Material downloaded from myCBSEguide.com. 15 / 25


125
6.9 cm.
20.

Let A (8,6) , B (8, 2) and C ( 2, 2) be the vertices of given the triangle and Let P( ) be
the circumference of the triangle
Then PA = PB = PC

Taking

Again, taking

x = 5
Coordinates of P are (5, 2)

Radius PA

21.

Pocket Money Number of Cumulative


(In Rupees) Employees (f) Frequency

Material downloaded from myCBSEguide.com. 16 / 25


126
5-10 3 3

10-15 6 9

15-20 9 18

20-25 11 29

25-30 14 43

30-35 10 53

35-40 7 60

This lies in class 25-30


Median group in 25-30
Now using formula

where



Rs.

Or

Since the maximum frequency = 8 and it corresponds to the class 3-5


Modal class = 3-5
Here, l = 3, h = 2, f1 = 8, f0 = 7, f2 = 2

We know that mode Mo is given by

= 3 + 0.2857 = 3.286 nearly.


22. There are 30 discs in the box out of which one disc can be drawn in 30 ways.
Total number of elementary events 30
i. From number 1 to 30, there are 21 two digit numbers, namely 10,11,12….30 out of

Material downloaded from myCBSEguide.com. 17 / 25


127
these 21 two digit numbered disc can be chosen in 21 ways.
Favourable number of elementary events 21
Hence P (getting a to digit numbered disc)

ii. Those numbers from 1 to 30 which are perfect square 1,4,9,16,25 i.e. squares of 1,
2,3,4,5 respectively. Therefore, there are 5 disc marked with the number which are
perfect squares.
Favourable number of elementary events 5
Hence, P (Getting a disc marked with a number which is perfect square)
23. Proof : Let us assume, to the contrary that is rational.
So, we can find integers r and such that
Suppose r and s have a common factor other than 1. Then we divide by the common
factor to get , where a and b are coprime.
So, we can write for some integer c.
Substituting for a we get , that is, .
This mean that 2 divides and so 2 divides b (again using theorem 1.3 with p =2 ).
Therefore, a and b have at least 2 as a common factor.
But this contradict the fact that a and b have no common factor other than 1.
This contradiction has arisen because of our incorrect assumption that is rational.
So, we conclude that is irrational.

Or

According to Euclid’s division lemma,


Given positive integers a and b, there exist unique integers q and r satisfying
a = bq + r, 0 ≤ r < b.
Let a be any positive integer and b = 3.
Then, let a = 3q + r, where q > 0 and 0< r < 3
a = 3q or a = 3q + 1 or a = 3q +2
We have three cases :

i. when a = 3q, then, a3 = (3q)3 = 9(3q3) = 9k [where k = 3q3 is an integer]


ii. when a = 3q + 1,

then, a3 = (3q + 1)3 = 9(3q3) + 9(3q2) + 9q + 1

Material downloaded from myCBSEguide.com. 18 / 25


128
= 9(3q3 +3q2 +3q) + 1 = 9k1 + 1 [where k1 = 3q3 + 3q2 + 3q is an integer]

iii. When a = 3q + 2

then, a3 = (3q + 2)3 = 9(3q3) + 9(6q2) + 9(4q) + 8 = 9(3q3 + 6q2 + 4q) + 8 = 9k2 + 8

[where k2 = 3q3 + 6q2 + 4q is an integer]


Therefore, the cube of any positive integer is of the form 9m, 9m + 1 or 9m + 8.
24. Dividend

and Divisor

So quotient
Remainder
Now Divisor quotient + remainder

Dividend
In this way, the division algorithm is verified.
25. Let the cost of one bat Rs.
And cost of one ball Rs.
Algebraically form

Solution by graphical method:

Material downloaded from myCBSEguide.com. 19 / 25


129
Table

x 200 100 0

y 50 100 150

x 225 100 75

y 25 75 125

Plot the points and draw the lines passing through them to represent the equations as
shown in figure.

The two lines intersect at point ( 150, 75)


So is the required solution of the pair of linear equation
Cost of one bat = Rs. 150 & Cost of one ball = Rs. 75
Cost of one bat & one ball 150+75 = Rs. 225

Material downloaded from myCBSEguide.com. 20 / 25


130
Or

Let the required numbers be x and y.


Then, x + y = 16
And,

xy = 48
We can write

x – y =

∴ x + y = 16…(i)
x – y = 8…(ii)
Or, x + y = 16…(iii)
x – y = –8…(iv)
On solving (i) and (ii), we get x = 12 and y = 4
On solving (iii) and (iv), we get x = 4 and y = 12
Thus, the required numbers are 12 and 4.
26.

BE and CF are medians of the ABC in which


From ABC,
(Pythagoras Theorem)
From ABE,

or (E is midpoint of AC )

or

Material downloaded from myCBSEguide.com. 21 / 25


131
or
From FAC,

or ( F is the mid-point of AB)

or
or
Adding and we have

i.e. [from (i)]


27. L.H.S.

R.H.S.
28.

OE AB

BE AB

Material downloaded from myCBSEguide.com. 22 / 25


132
m
Area of segment

Area of two segment


Area of square lawn

Sum of the areas of the lawn and the flower beds

Ans.
29. Radius of cylinder m.
Height of cone m
and height of cylinder m

Capacity of the tent = Vol. of cone + Vol. of cylinder

Material downloaded from myCBSEguide.com. 23 / 25


133
Curved surface area of tent = Curved surface area of cone + Curved surface area of
cylinder

Cost of canvas per Rs. 100


Cost of canvas 75.9 Ans.
30.

Class - interval Frequency (f) c.f.

0 100 2 2

100 200 5 7

200-300 x 7 + x

300 - 400 12 19 + x

400 - 500 17 36 + x

500 - 600 20 56 + x

600 - 700 y 56 + x+ y

700 - 800 9 65 + x +y

800- 900 7 72 + x + y

900 - 1000 4 76 + x + y

Given n = 100

or
Median is 525 which lies interval 500-600

Median

525 = 500

Or

Material downloaded from myCBSEguide.com. 24 / 25


134
Or
Or

From equation

Or

Marks Cf Marks Cf

Less than 5 4 More than 0 100

Less than 10 10 More than 5 96

Less than 15 20 More than 10 90

Less than 20 30 More than 15 80

Less than 25 55 More than 20 70

Less than 30 77 More than 25 45

Less than 35 95 More than 30 23

Less than 40 100 More than 35 5

Material downloaded from myCBSEguide.com. 25 / 25


135
CBSE Class X Mathematics
Sample Paper 08

Time allowed: 3 Hours Max. Marks: 80


General Instructions:

i. All questions are compulsory.


ii. The question paper consists of 30 questions divided into four sections A, B, C and D.
iii. Section A contains 6 questions of 1 mark each. Section B contains 6 questions of 2 marks
each. Section C contains 10 questions of 3 marks each. Section D contains 8 questions of 4
marks each.
iv. There is no overall choice. However, an internal choice has been provided in four
questions of 3 marks each and three questions of 4 marks each. You have to attempt only
one of the alternatives in all such questions.
v. Use of calculators is not permitted.

SECTION – A

1. The sum and product of zeros of a quadratic polynomial are and –7 respectively.
Write the polynomial ?
2. Can two positive integers have their H.C.F and L.C.M as 12 and 512 respectively ? Justify.
3. If , then determine the value of .
4. Write the relation between Mean, Mode and Median.
5. If the straight line joining two points P (5, 8) and Q (8, k) is parallel to x-axis, then write
the value of k.
6. A tangent PQ at a point P of a circle of radius 5 cm meets a line through the centre O at a
point Q so that OQ = 12 cm. Write the length of PQ .

SECTION – B

7. The 7th term of an A.P. is – 4 and its 13th term is – 16. Find the sum of its first 19 terms.
8. If the points (4, 3) and (x, 5) lie on the circumference of the circle whose centre is (2, 3),
then find the value of x.
9. Show that is irrational.

Material downloaded from myCBSEguide.com. 1 / 15


136
10. In Fig-1, if EF||BC and FG||CD, prove that, .

11. A quadrilateral ABCD is drawn to circumscribe a circle (fig-2).


Prove that, AB + CD = AD + BC.

12. From a solid cylinder whose height is 2.4 cm and diameter 1.4 cm, a conical cavity of the
same height and same diameter is hollowed out (fig-3). Find the total surface area of the
remaining solid.

SECTION – C

13. Find the roots of the equation 3x2 – 7x – 2 = 0 by the method of completing the square.
14. Solve the pair of linear equations 8x + 5y = 9 and 3x + 2y = 4 by cross-.multiplication
method.
15. Poved that if in two triangles, sides of one triangle are in the same ratio of the sides of the
other triangle, then their corresponding angles are equal.
16. Prove that the points A(– 5, 4), B(–1, –2) and C(5, 2) are the vertices of an isosceles right-
angled triangle.

Or

The vertices of a triangle are A (-1, 3), B (1, -1) and C (5, 1). Find the length of the median
through the vertex C.
17. Cards marked with numbers 3, 4, 5, …, 50 are placed in a box and mixed thoroughly. One

Material downloaded from myCBSEguide.com. 2 / 15


137
card is drawn at random from the box. Find the probability that number on the drawn
card is a two digit number which is a perfect square.

Or

A die is thrown once. Find the probability of getting (i) an even number (ii) a number
greater than 3 (iii) a composiite number
18. A die is thrown once. Find the probability of getting (i) a prime number; (ii) an odd
number.
19. Solve for x :

Or

If the roots of the equation (b – c)x2 + (c – a)x + (a – b) = 0 are equal,


then prove that 2b = a + c.
20. If , then prove that,
21. Two tangents PA and PB are drawn to a circle with centre O from an external point P.
Prove that ∠ APB = 2 ∠ OAB.

Or

A circle with centre O, diameter AB and a chord AD is drawn. Another circle is drawn
with AO as diameter to cut AD at C. Prove that BD = 2OC.
22. State the ‘Fundamental Theorem of Arithmetic’ . Use Euclid’s division algorithm to find
the HCF of 196 and 38220. Hence find the LCM of these numbers.

SECTION – D

23. Draw a pair of tangents to a circle of radius 5 cm which are inclined to each other at an
angle of 60°.

Or

Material downloaded from myCBSEguide.com. 3 / 15


138
Draw a line segment AB of length 8 cm. Taking A as centre, draw a circle of radius 4 cm.
and constant the pair of tangents of the circle from point B and measure their lengths.
24. Check graphically whether the pair of equations x + y = 8 and x – 2y = 2 is consistent. If so,
solve them graphically. Also find the coordinates of the points where the two lines meet
the y-axis.
25. If and show that,

Or

If prove that
26. If the median of the distribution given below is 28.5, find the values of x and y.The sum of
all frequency is 60

Class-interval 0-10 10-20 20-30 30-40 40-50 50-60

Frequencry 5 x 20 15 y 5

27. Find the area of the shaded region in fig-5, where ABCD is a square of side 20cm.

28. The angle of elevation of a cloud from a point 60 m above a lake is 30o and the angle of

depression of the reflection of the cloud in the lake is 60o. Find the height of the cloud.

29. A metallic right circular cone 20 cm high and whose vertical angle is 60o is cut into two
parts at the middle of its height by a plane parallel to its base. Find the volume of the
frustum so obtained.
30. The term and the sum of first n terms of an A.P are respectively are and and
. Prove that, .

Or

Find the sum of first 40 positive integers divisible by 6. Also find the sum of first 20
positive integers divisible by 5 or 7.

Material downloaded from myCBSEguide.com. 4 / 15


139
CBSE Class X Mathematics
Sample Paper 08
Solutions

SECTION – A

1. Sum of zeroes =

Product of zeroes =

Coefficient of x2 = 5 and coefficient of x = 1 and the term free from x = - 35


The polynomial is
2. No. There can not exist two numbers satisfying the given condition, because here L.C.M
(= 512) is not divisible by H.C.F ( = 12).
3. i.e.,

4. 3 Median = Mode + 2 Mean


5. k = 8
6.

7. 7th term, . By question, a + 6d = – 4 … (i)

13th term, . By question, a + 12d = – 16 … (ii)


Now sum of first 19 terms
=
[Adding (i) and (ii), we get, 2a+ 18d = – 16 ]

8. Length of radius =
i.e.,

i.e., (x – 2)2 = 0
i.e., x = 2
Hence, value of x = 2.

Material downloaded from myCBSEguide.com. 5 / 15


140
9. If possible, let us assume that is rational and equals to
i.e., , where a and b are positive integers prime to each other and b >1

i.e., ….. (i)


From (i), we see that, is not an integer, as a and b are prime to each other, so
are also prime to each other, but 3b is an integer
i.e in (i), a fraction equals to an integer, which contradicts our initial assumption.
Hence, is irrational. 1
10. Since, EF||BC,
Since, FG||CD,

By (i) and (ii),

11. Since, the lengths of tangents drawn from an external point to a circle are equal.

AP = AS … (i) BP = BQ … (ii)
CQ = CR … (iii) DR = DS … (iv)
Now, AB + CD
= AP + PB + CR + RD
= AS + BQ + CQ + DS
= (AS + DS) + (BQ + CQ)
= AD + BC

Material downloaded from myCBSEguide.com. 6 / 15


141
Hence proved.
12. Height of the solid cylinder (h)= 2.4 cm
Diameter of its base (2r)= 1.4 cm.

Therefore its base radius (r)= 0.7 cm


Height and diameter of the conical cavity are equal to those of the cylinder.
Remaining surface area
= (Curved surface area( outside) of cylinder) + (surface area of its bottom) + ( curved
surface area of the conical cavity)
=

13.
i.e., [multiplying both sides by 3 ]

i.e.,

i.e.,

i.e.,

i.e.,

i.e.,

i.e.,

Therefore, the roots are and


14.

Solving equations (i) and (ii) by cross-multiplication method, we get,

Material downloaded from myCBSEguide.com. 7 / 15


142
i.e.,
i.e.,
i.e.,
Hence the solutions are x = – 2 , y = 5.
15. Let ABC and DEF be two triangles such that,
.

To prove that,

From AB and AC cutting AP = DE, AQ = DF let us join P and Q

so,

PQ || BC [If a line divides any two sides of a triangle in the same ratio, then the line is
parallel to the third side.]
So, (common angle)
Therefore, DABC and DDEF are equiangular and so their corresponding sides are in the
same ratio.
Hence, i.e.,

So
i.e., PQ = EF
DEF APQ (S-S-S)
So

Material downloaded from myCBSEguide.com. 8 / 15


143
Hence the corresponding angles of the triangles are equal.

16. Length of units,

Length of units,

Length of units,

Here we get,
Hence the triangle is an isosceles triangle.

Also, we observe that

Therefore, by Pythagoras theorem, DABC is a right-angled triangle (right angle at B)


Hence the triangle is an isosceles right-angled triangle.
17. According to the question, cards are mixed thoroughly and one card is drawn at random
from the box, so the event of drawing a card is equally and likely.
Since cards are marked with numbers 3, 4, 5, …, 50 ,
So there are 48 cards
Here the total number of possible outcomes = 48.
Let the event of drawing a card at random bearing two digit perfect square number be E.
Then the number of outcomes favourable to the event E = 4
(here two digit perfect numbers are 16, 25, 36, 49 )
Therefore,

18. When a die is thrown once, then the number of total outcomes = 6
Let the E be the event of getting one prime number.
Here prime numbers are 2, 3, 5
Then the number outcomes favourable to E = 3
Hence,

Again let F be the event of getting an odd number.


Here odd numbers are 1, 3, 5
Hence,

19. (x 1, 2, 3)

i.e., ( given x 1, 2, 3)

Material downloaded from myCBSEguide.com. 9 / 15


144
i.e.,
i.e.,
i.e.,
i.e., x = 0 or x = 4
Hence the solutions are x = 0 and x = 4.
20. Given,
i.e.,
i.e., [multiplying both sides by ]
i.e.,
i.e.,
i.e.,
Hence proved.
21. According to the question, from an outside point P two tangents PA and PB are drawn to a
circle with centre O (fig-4).

To prove that, ∠APB = 2 ∠ OAB.


Since the lengths of tangents drawn from an external point to a circle are equal.
So, PA = PB. i.e., PAB is isosceles.

[since, radius through point of contact is perpendicular to the tangent at the point of
contact]
Hence proved.
22. Fundamental Theorem of Arithmetic :
Every composite number can be expressed ( factorised) as a product of primes, and this
factorisation is unique, apart from the order in which the prime factors occur.
Since 867 > 255, we apply the division lemma to 867 and 255, to get
867 = 255 3 + 102
255= 102 2 + 51

Material downloaded from myCBSEguide.com. 10 / 15


145
102= 51 2 + 0
The remainder is 0 (zero) and divisor is 51.
Therefore the HCF of 867 and 255 is 51.
Hence LCM of 867 and 255
=
23. Construct a circle of radius 5 cm.
Let its centre be O.

Now construct one radius (say OP) of the circle.


At O, draw an angle of 60°,
and draw a perpendicular to OP at P, 90°.
Extend these lines to meet at T (say).
Now, taking T as centre and a radius equal to TP
draw an arc which cuts the circle at Q. Join T, Q.
Hence TP and TQ are the two tangents to the given circle of radius 5 cm
which are inclined to each other at an angle of 60°
[For justification of the construction:
i.e., ]

24.

x 0 4 8

y = 8 –x 8 4 0

Three solutions for equation (i)are given in the table :

Three solutions for equation (ii) are given in the table :

x 0 2 8

–1 0 3

Material downloaded from myCBSEguide.com. 11 / 15


146
Drawing Line AC
Drawing Line PR

Plotting points A(0, 8), B(4, 4) and C(8, 0) on graph paper the straight line AC is obtained as
graph of the equation
(i) Plotting points P(0, –1), Q(2, 0) and R(8, 3) on graph paper the straight line PR is
obtained as graph of the equation
(ii) From the graph, it is clear that a point M(6, 2) common to both the lines AC and PR.
So the pair of equations is consistent and the solutions of the equations are x = 6 and y =
2.
From the graph it is seen that the coordinates of the points where the lines AC and PR
meets the y-axis are (0, 8) and (0, –1) respectively.
25.
Given, … (i)
and … (ii)
adding (i) and (ii), we get,
and subtracting (ii) from (i), we get,

Material downloaded from myCBSEguide.com. 12 / 15


147
. Hence proved.
26.

Class interval Frequency Cumulative Frequency

0-10 5 5

10-20 x 5+x

20-30 20 25+x

30-40 15 40+x

40-50 y 40+x+y

50-60 5 45+x+y

Total 60

It is given that, n = 60
i.e., 45 + x + y = 60 i.e., x + y = 15
The median is 28.5, which lies in the class 20-30
So, l = 20, f = 20, cf = 5+x, h = 10

We know,

Here,

Therefore y = 15 – 8 = 7.
27. Let the square be ABCD of side 20 cm.
Area of the square ABCD = .

Diameter of each circle (in fig-5) =


Therefore radius of each circle = 5 cm.
So area of each circle =

Material downloaded from myCBSEguide.com. 13 / 15


148
Total area of four squares =
Hence area of the shaded region in the fig-5

28. Let in the adjacent figure EC be the surface of water in the lake.
A is the position of the observer. AE = 60 m.
Also let B is the position of cloud and D be its image for fig.

In the lake and BF = h metre


So, BC = CD = (h + 60)m (see fig)
FC = 60 m.
By question,
In ABF,

In AFD,
[ since DF = DC + CF]

By (i) and (ii),

Hence height of the cloud from the water surface of the lake
=BC = 60 + 60 m = 120 m
29. Let ADH be a metallic right circular cone, whose height is 20 cm. is cut into two parts at
the middle of its height by a plane parallel to its base. The frustum is EBFHCDE
Given that, ,

Material downloaded from myCBSEguide.com. 14 / 15


149
AC = 20 cm 1
According to question, AB = BC = 10 cm.
In ABF,

Again in In ACH,

Therefore volume of the frustum EBFHCDE =


= [here H = AC = 20 cm AB = 10 cm]

Hence the required volume = 2444.44 cm3


30. Let first term and the common difference of the AP be a and d respectively.
Then and
According to question,

This is an identity.
Now putting, n = 2n – 1 and m = 2m– 1, we get,

Hence proved.

Material downloaded from myCBSEguide.com. 15 / 15


150
CBSE Class X Mathematics
Sample Paper 09

Time allowed: 3 Hours Max. Marks: 80


General Instructions:

i. All questions are compulsory.


ii. The question paper consists of 30 questions divided into four sections A, B, C and D.
iii. Section A contains 6 questions of 1 mark each. Section B contains 6 questions of 2 marks
each. Section C contains 10 questions of 3 marks each. Section D contains 8 questions of 4
marks each.
iv. There is no overall choice. However, an internal choice has been provided in four
questions of 3 marks each and three questions of 4 marks each. You have to attempt only
one of the alternatives in all such questions.
v. Use of calculators is not permitted.

SECTION – A

1. Find the zeroes of the quadratic polynomial .

2. A ladder of length H metre makes an angle of 60o with the ground when placed against a
wall. If the distance between the feet of the ladder and the wall be 5 m, then determine H.
3. Captain of a team tosses two different coins, one golden colour and other of silver colour
simultaneously. What is the probability that he gets atleast one head ?
4. Write the sum of the first 100 natural numbers.
5. Find the coordinate of the mid-point of the line segment joining the points whose

coordinates are and .

6. From an outside point A, two tangents AB and AC are drawn to touch the circle with

centre at O (in fig-1). Given BAC = 30o. Find AOB.

Material downloaded from myCBSEguide.com. 1 / 17


151
SECTION – B

7. Find the values of y for which the distance between the points P(2, – 3) and Q(10, y) is 10
units.
8. Find the discriminant of the equation and hence find the nature
of its roots.
9. The sum of first 30 and 40 terms of an AP are respectively 2265 and 4020, then find the
common difference of the AP.
10. The height and base diameter of a solid cylinder are 16 m and 2r m respectively. The
cylinder is melted and recast to 12 solid spheres of same base diameter. Find r.
11. Two concentric circles are of radius 7 cm and cm. Find the length of the chord of the
bigger circle, which touches the smaller circle.

12. If ABC and DEF are two similar triangles and their areas are 81 cm2 and 144 cm2
respectively. The bases of the triangles are respectively BC and EF. If EF = 15 cm, find BC.

SECTION – C

13. The denominator of a fraction is one more than thrice of the numerator. If the sum of the
fraction and its reciprocal is , then find the fraction.
14. Use Euclid’s division lemma to show that the cube of any positive integer is of the form
9m, 9m + 1 or 9m + 8.
15. An army contingent of 616 members is to march behind an army band of 32 members in
a parade. The two groups are to march in the same number of columns. What is the
maximum number of columns in which they can march?

16. The ratio of the sum of n-terms of two APs is (2n + 4) : (5n +2), find the ratio of their 12th
terms.

Or

Find the sum of all 3 digit numbers which leave remainder 3 when divided by 5.

17. The vertices of a ABC are A(4, 9), B and C(9, 4). A line is drawn to intersect

sides AB and AC at P and Q respectively, such that . Find the area of

APQ.

Or

Material downloaded from myCBSEguide.com. 2 / 17


152
The vertices of a ABC is (1, 2), (3, 1) and (2, 5). Point D divides AB in the ratio 2:1 and P
is the mid-point of CD. Find the coordinates of the point P.
18. In fig-2, ABC and ABD are on the same base AB and on opposite sides of AB. If CD
intersects AB at O, then show that,

19. In fig-3, PQ and RS are two parallel tangents to a circle with centre O and another tangent
EF with point of contact C intersecting PQ at E and RS at F.
Prove that ∠EOF = 90°.

Or

Prove that, the lengths of tangents drawn from an external point to a circle are equal.
20. Prove that: .
21. A bag contains 5 red ball and some blue balls. If the probability of drawing a blue ball is
double that of a red ball, determine the number of blue balls in the bag.

Or

A bag contains 8 red balls and x blue balls. the odd against drawing a blue ball are 2:5.
what is the value of x?

Material downloaded from myCBSEguide.com. 3 / 17


153
22. One card is drawn from a well-shuffled deck of 52 cards. Find the probability of getting :
a. a king of red colour
b. a spade
c. a face card

SECTION – D

23. If we add 1 to the numerator and subtract 1 from the denominator, a fraction reduces to
1. It becomes if we only add 1 to the denominator. What is the fraction ?

Or

The sum of a 2 digit number and number obtained by reversing the order of the digits is
99. If the digits of the number differ by 3. Find the number.

24. Find the common difference of an A.P. whose 1st term is 100 and the sum of whose first
six terms is 5 times the sum of the next six terms.
25. Draw a circle with the help of a bangle. Take a point outside the circle. Construct the pair
of tangents from this point to the circle.

Or

Let ABC be a right triangle in which AB = 6 cm, BC = 8 cm and ∠ B = 90°. BD is the


perpendicular from B on AC. The circle through B, C, D is drawn. Construct the tangents
from A to this circle.
26. Prove that,

27. Find the area of the segment APB shown in fig-4, if radius of the circle is 14 cm,
and (use )

Material downloaded from myCBSEguide.com. 4 / 17


154
28. There are two poles either on each bank of a river, just opposite to each other. One pole is
60 metre high. From the top of this pole, the angles of depression of the top and the foot

of the other pole are 30o and 45o respectively. Find the width of the river and the height
of the other pole.
29. In fig-5, a toy is in the form of a cone mounted on a hemisphere of common base of radius
7 cm. The total height of the toy is 31 cm. Find the total surface area of the toy. [ ]

30. The following frequency distribution gives the monthly consumption of electricity of 68
consumers of a locality. Find the median and mode of the data.

Monthly
65-85 85-105 105-125 125-145 145-165 165-185 185-205
Consumption

Number of
4 5 13 20 14 8 4
Consumer

Or

If the mean of the following distribution is 27, find the value of p. Also find the median
and mode.

Class-interval 0-10 10-20 20-30 30-40 40-50

No of workers 8 p 12 13 10

Material downloaded from myCBSEguide.com. 5 / 17


155
CBSE Class X Mathematics
Sample Paper 09
Solutions

SECTION- A

1. 3x2 + 7x + 2

the zeroes are .


2.

Therefore, H = 10 m.

3. Total outcomes = 22 = 4

4. Sum of the first 100 natural numbers =

5. Coordinates of the required mid-point = i.e.,

6. Since tangents drawn from an external point to a circle subtend equal angles at the
centre of the circle.

Material downloaded from myCBSEguide.com. 6 / 17


156
7. By question,

i.e.,
i.e.,
i.e.,
i.e.,

Hence values of y are 3, – 9

8. Discriminant of is = 80 - 80 = 0

Therefore roots of the given quadratic equation are real and equal.
9. We know that the sum of first n-terms of an AP (whose first term is and common
difference is d) is
By question,

Again by question,

Subtracting (i) from (ii), we get,


10d = 50 i.e., d = 5
Hence the common difference of the AP is 5.
10. The volume of the solid cylinder =

Volume of each sphere =


[since its base diameter is equal to that of the cylinder]
By question, . Hence r = 1 m
11. Let C is the centre of two concentric circles of radii 5cm and 3 cm.

Let AB, a chord of the bigger circle which touches the smaller circle at M.
CM = cm, CA = 7 cm
[radius through point of contact is to tangent]
[by Pythagoras Theorem]

Material downloaded from myCBSEguide.com. 7 / 17


157
i.e.,
Therefore, AB = 2AM = 2´6 = 12 cm.
[Line segment, drawn from centre of a circle perpendicular to any chord, bisects the
chord ]
Hence, the length of the chord is 12 cm.
12. We know that, the ratio of the areas of two similar triangles is equal to the square of the
ratio of their corresponding sides.
Therefore for the given problem,
Hence, BC = 11.25 cm
13. Let the fraction be
According to first condition, y = 3x + 1.
So the fraction becomes

Again by the 2nd condition,

Therefore. x = 5 (since, x >0)


Hence the required fraction =
14. According to Euclid’s division lemma,
Given positive integers a and b, there exist unique integers q and r satisfying
a = bq + r, 0 ≤ r < b.
Let a be any positive integer and b = 3.
Then, let a = 3q + r, where q > 0 and 0< r < 3
a = 3q or a = 3q + 1 or a = 3q +2
We have three cases :

i. when a = 3q, then, a3 = (3q)3 = 9(3q3) = 9k [where k = 3q3 is an integer]


ii. when a = 3q + 1,

then, a3 = (3q + 1)3 = 9(3q3) + 9(3q2) + 9q + 1

Material downloaded from myCBSEguide.com. 8 / 17


158
= 9(3q3 +3q2 +3q) + 1 = 9k1 + 1 [where k1 = 3q3 + 3q2 + 3q is an integer]

iii. When a = 3q + 2

then, a3 = (3q + 2)3 = 9(3q3) + 9(6q2) + 9(4q) + 8 = 9(3q3 + 6q2 + 4q) + 8 = 9k2 + 8

[where k2 = 3q3 + 6q2 + 4q is an integer]


Therefore, the cube of any positive integer is of the form 9m, 9m + 1 or 9m + 8.
15. To find the maximum number of columns in which members can march,
we have to calculate the HCF of 32 and 616.
Using Euclid’s algorithm,
616 = 32 19 + 8,
32 = 8 4 + 0
Therefore the HCF of 32 and 616 is 8
Hence they can march in 8 columns each.
16. Let, be the first terms of the two APs also let be their common
differences respectively.
According to question,

This is an identity.
Putting n = 23, we get,

i.e.,

i.e.,

i.e.,

i.e.,

Hence ratio of their 12th terms is 50 : 117.


17. Since P divides AB internally in the ratio 2 : 3,

Material downloaded from myCBSEguide.com. 9 / 17


159
So, abscissa i.e., x-coordinate of P =

and ordinate i.e., y-coordinate of P =


Therefore coordinates of P are (3, 5)
Since, Q divides AC internally in the ratio 2 : 3
Again, abscissa i.e., x-coordinate of Q =
and ordinate i.e., y-coordinate of Q =
Therefore coordinates of Q are (6, 7)
Using, formula: ,
area of APQ =

18. Given that, ABC and ABD are on the same base AB but on opposite sides of AB.

To prove that,

Construction :Let CM AB and DN AB are drawn.


Proof: In CMO and DNO,

CMO = DNO = 90o


COM = DON [ vertically opposite ]
CMO ~ DNO [by A-A property ]
So,
… (ii) and
… (iii)

From (ii) and (iii) , Hence proved.

19. Given, two tangents PQ and RS, which touch the circle, with centre O, at points A and B.
Also, PQ || RS.

Material downloaded from myCBSEguide.com. 10 / 17


160
Another tangent EF touches the circle at C and meets the lines PQ and RS at E and F
respectively.
To prove, .
Construction : O, C joined.
Proof : .
[Since, radius through point of contact is perpendicular to the tangent]
In

OA = OC [radius of same circle]


EA = EC [Tangents drawn from outside point to a circle are equal]

i.e.,
Similarly, it can be shown that,

Since PQ || RC and EF is a transversal,


So,

. Hence proved.
20.

Material downloaded from myCBSEguide.com. 11 / 17


161

= RHS
Hence proved.
21. Let the number of blue balls in the bag be x.
Therefore total number of balls in the bag = 5 + x
Let probability of drawing a blue ball be P(blue).
Let probability of drawing a red ball be P(red).
Then,
P(blue)

and
P(red)

According to the question, P(blue) = 2´ P(red)


i.e.,
i.e., x=10 [since x can not be negative ]
Hence number of blue balls in the bag is 10.
22. Total number of favourable outcomes = 52.
a. We know that, cards of diamond and heart are of red colour.
There exist one king of Diamond and one king of heart. Number of outcomes
favourable to ‘getting a king of red colour’ = 2
P(getting a spade)

b. A well-shuffled deck of 52 cards contains 13 cards of spade.


So, number of outcomes favourable to ‘getting a spade’ = 13.
P(getting a king of red colour)

Material downloaded from myCBSEguide.com. 12 / 17


162
c. A well-shuffled deck of 52 cards contains 12 face cards.
So, number of outcomes favourable to ‘getting a face card’ = 12.
P(getting a face card)

23. Let the numerator and the denominator be x and y respectively.


i.e., the fraction is
According to question we get, and

Now,
and
Substituting y in (ii) with the help of (i),
2x = x + 2 +1
i.e., x = 3 1
Now from (i), y = 3 +2 =5.
Hence the fraction is .

24. Let the 1st term and common difference of the AP be a and d respectively.
Here a = 100.
We know that sum of first n-terms of an AP, .
Now sum of first 6-terms of the AP is
.
and sum of next 6-terms of the AP
= S12 - S6

= 6(200 + 11d) - 3(200 + 5d)


= 1200 + 66d - 600 - 15d = 600 + 51d
According to the question,
600+15d = 5(600+51d)
i.e., 600 + 15d = 3000 + 255d
i.e., 600 – 3000= 255d –15d

Material downloaded from myCBSEguide.com. 13 / 17


163
i.e., 240d = – 2400
d = – 10
Hence the common difference of the AP is (– 10).
25.

Steps of construction :
Draw a circle with help of a bangle.
Take two non parallel chords AB and CD
Draw perpendicular bisectors of these chords.
Mark point O where two bisectors intersect. Point O is center of the circle.
Take a point P outside the circle and join PO.
Bisect PO. Let M be mid-point of PO.
Taking M as centre and MO as radius, draw a circle.
Let it intersect the given circle at the points Q and R.
Join PQ and PR.
26.

Hence proved.
27. Area of the segment APB = Area of the sector OAPBO – area of OAB.

Material downloaded from myCBSEguide.com. 14 / 17


164
Here area of the sector OAPBO
=

Let, OM AB is drawn. Given,


Since, perpendicular drawn from centre of a circle to a chord bisects the chord,
So, here,

Now

Therefore area of OAB =

Hence, Area of the segment APB


= Area of the sector OAPBO – area of OAB

=
28. Let two poles AB(= 60 m) and PQ be either on each bank of a river.

Let PQ = h m. and width of the river i.e., BQ = x m.

Material downloaded from myCBSEguide.com. 15 / 17


165
In figure, PR AB. AR = AB – BR = AB – PQ = (60 – h) m
Acc to question,
Angle of depression of P from A
= XAP = APR (alternate angle) =
and angle of depression of Q from A
= XAQ = AQB (alternate angle) =
Now in AQB,
i.e., i.e., x =60
Now in APR,

i.e.,

i.e., [since x = 60]

i.e.,

i.e.,

Hence the width of the river is 60 m and height of the other pole is 25.36 m.
29. Total height of the toy is 31 cm i.e., in fig-5, AD = 31 cm.

Radius of the common base is 7 cm.


i.e., OC = OB = OD = 7 cm.
AO = AB – OD = 31 – 7 = 24 cm.
Surface area of the toy
= slant surface area of the cone + curved surface area of the hemisphere
Here, slant surface area of the cone = sq. unit

Curved surface area of the hemisphere


=

Material downloaded from myCBSEguide.com. 16 / 17


166
Hence the total surface area of the toy =( 550 + 308) cm2 = 858 cm2.
30. For table = 1

Monthly consumption (in units) Number of consumer (fi) Cumulative frequency

65-85 4 4

85-105 5 9

105-125 13 22

125-145 20 42

145-165 14 56

165-185 8 64

185-205 4 68

Total N=68

For Median.
Here, , then , which lies in the interval 125-145.
Median class is 125-145.
Here, l = 125, n = 68, f = 20, cf = 22 and h = 20.

For Mode :
In the given data, maximum frequency is 20 and it corresponds to the class interval 125-
145
Modal class = 125-145
and = 125, f1 = 20, f0 = 13, f2 = 14 and h = 20

Now,

Hence Median= 137 units, Mode = 135.77 units.

Material downloaded from myCBSEguide.com. 17 / 17


167
CBSE Class X Mathematics
Sample Paper 10

Time allowed: 3 Hours Max. Marks: 80


General Instructions:

i. All questions are compulsory.


ii. The question paper consists of 30 questions divided into four sections A, B, C and D.
iii. Section A contains 6 questions of 1 mark each. Section B contains 6 questions of 2 marks
each. Section C contains 10 questions of 3 marks each. Section D contains 8 questions of 4
marks each.
iv. There is no overall choice. However, an internal choice has been provided in four
questions of 3 marks each and three questions of 4 marks each. You have to attempt only
one of the alternatives in all such questions.
v. Use of calculators is not permitted.

SECTION - A

1. The decimal expansion of the rational number will terminate after how many places

of decimal?
2. Find the nature of the roots of the equation .
3. A boy walks 12m due east and 5 m due south. How far is he from the starting point?
4. If the point C(k, 4) divides the join of points A(2, 6) and B(5, 1) in the ratio 2 : 3, then find
the value of k.
5. Find the area of annulus whose inner and outer radii are 6 cm and 8 cm.
6. In a family of three children, find the probability of having at least one boy.

SECTION - B

7. Check whether on simplification gives a rational or irrational number.

8. Gunal saved Rs.10 in the first week of a month and then increased his weekly savings by
Rs.2.75. If in the nth week, his savings become Rs.59.50, find ‘n’.
9. Without using trigonometric tables prove that:
tan 1° tan 11° tan 21° tan 69° tan 79° tan 89° = 1

Material downloaded from myCBSEguide.com. 1 / 19


168
10. Find the third vertex of the triangle ABC. If two of its vertices are at A(-3, 1) and B(0, 2)

and the mid-point of BC is at D

11. D and E are points on the sides AB and AC respectively of a ABC. If AD = 5.7 cm, DB =
9.5 cm, AE = 4.8 cm and EC = 8 cm, then determine whether DE || BC or not.
12. Three cubes each of side 15 cm are joined end to end. Find the total surface area of the
resulting cuboid.

SECTION - C

13. Show that any positive odd integer is of the form (4m + 1 ) or ( 4m + 3), where m is some
integer.

14. If two zeroes of the polynomial x4 + 3x3 - 20x2 - 6x + 36 are and - , then find the
other zeroes of the polynomial.
15. Solve:

16. A class consists of a number of boys whose ages are in A.P., the common difference being
4 months. If the youngest boy is just eight years old and if sum of the ages is 168 years,
then find the number of boys in the class.

OR

Find for the AP in -9, -14, -19, -24…


17. Find the coordinates of the centroid of a triangle whose vertices are A( ),

OR

In figure, ABC is a right triangle right-angled at B. Medians AD and CE are of respective


lengths 5 cm and , find length of AC.

18. If then show that

Material downloaded from myCBSEguide.com. 2 / 19


169
19. Prove that:

OR

If , show that

20. A square field and an equilateral triangular park have equal perimeters. If the cost of

ploughing the field at rate of Rs 5 perm2 is Rs 720, find the cost of maintaining the park at

therate of Rs 10 per m2.


21. The following table shows the marks obtained by 100 students of class X in a school
during a particular academic session. Find the mode of this distribution.

Less Less Less Less Less Less Less Less


Marks
than 10 than 20 than 30 than 40 than 50 than 60 than70 than 80

No of
7 21 34 46 66 77 92 100
students

OR

The percentage of marks obtained by 100 students in an examination are given below:

Marks 30-35 35-40 40-45 45-50 50-55 55-60 60-65

Frequency 14 16 18 23 18 8 3

Determine the median percentage of marks.

22. A bag contains 6 red balls and some blue balls. If the probability of drawing a blue ball
from the bag is double that of a red ball, find the number of blue balls in the bag.

SECTION - D

23. Solve the following system of linear equations graphically: 3x + y = 12 and x - 3y = -6.
Shade the region bounded by these lines and the x-axis. Also find the ratio of areas of
triangles formed by given lines with x-axis and the y-axis.

OR

A boat goes 30 km upstream and 44 km downstream in 10 hours. In 13 hours it can go 40

Material downloaded from myCBSEguide.com. 3 / 19


170
km upstream and 55 km down stream. Determine the speed of the stream and that of the
boat in still water.
24. A plane left 30 minutes late than its scheduled time and in order to reach the destination
1500 km away in time, it had to increase the speed by 250 km/h from the usual speed.
Find its usual speed.
25. Prove that the ratio of areas of two similar triangles is equal to the square of the ratio of
their corresponding sides. Use the above theorem, in the following. The areas of two

similar triangles are 81 cm2 and 144 cm2. If the largest side of the smaller triangle is 27
cm, find the largest side of the larger triangle.

OR

In a triangle if the square of one side is equal to the sum of the squares on the other two
sides. Prove that the angle apposite to the first side is a right angle. Use the above
theorem to find the measure of in figure given below.

26. Prove that the intercept of a tangent between two parallel tangents to a circle subtends a
right angle at the centre of the circle.
27. Construct a triangle similar to given ABC in which AB = 4 cm, BC = 6 cm and ABC = 60°,
such that each side of the new triangle is of given ABC.
28. The angle of elevation of the top a tower at a point on the level ground is 30°. After
walking a distance of 100m towards the foot of the tower along the horizontal line
through the foot of the tower on the same level ground, the angle of elevation of the top
of the tower is 60°. Find the height of the tower.
29. A container (open at the top) made up of a metal sheet is in the form of a frustum of a
cone of height 16 cm with radii of its lower and upper ends as 8 cm and 20 cm
respectively. Find
i. the cost of milk when it is completely filled with milk at the rate of Rs 15 per litre.

ii. the cost of metal sheet used, if it costs Rs 5 per 100 cm2

Material downloaded from myCBSEguide.com. 4 / 19


171
30. The median of the following data is 20.75. Find the missing frequencies x and y, if the
total frequency is 100.

Class-Interval 0-5 5-10 10-15 15-20 20-25 25-30 30-35 35-40

Frequency 7 10 x 13 y 10 14 9

OR

In the following distribution, locate the median mean and mode.

Monthly consumption of 65- 85- 105- 125- 145- 165- 185-


electricity 85 105 125 145 165 185 205

No. of Consumers 4 5 13 20 14 7 4

Material downloaded from myCBSEguide.com. 5 / 19


172
CBSE Class X Mathematics
Sample Paper 10
Solution

1.

The decimal expansion of will terminate after 4 places of decimal

2.

D = 108 > 0
Therefore the roots are real and distinct.
3.

Let the starting point be B.


Then BC =12 m & CA = 5m
By using Pythagoras theorem,

The distance between the starting point and end point = 13 m.


4.

By using section formula,

Material downloaded from myCBSEguide.com. 6 / 19


173
5. Area of Annulus =

6. Sample Space = {BBB, BBG, BGB, BGG , GBB, GBG, GGB,GGG}


n(S) = 8
P(having at least one boy) =

7.

= (a rational number)

8. a = 10 ; d = 2.75
an = 59.50

59.50 = 10 + (n – 1) (2.75)
n = 19

OR

a = -9
d = -14 - (-9) = -14 + 9 = -5
a30 - a20 = (a + 29d) - (a + 19d)

= 10d = 10 (-5) = -50


9. LHS = tan (90°-89°) tan (90°- 79°) tan (90°- 69°) tan 69° tan 79° tan 89°
= cot 89° cot 79° cot 69° tan 69° tan 79° tan 89°
= cot 89° cot 79° cot 69° × × ×

= 1
10. Let the third vertex be (x, y)

11. AD = 5.7 cm, DB = 9.5 cm, AE = 4.8 cm and EC = 8 cm

Material downloaded from myCBSEguide.com. 7 / 19


174
Since D and E are the points on AB and AC respectively.
Check whether
or not.

0.6 = 0.6
Therefore,
(each equal to 0.6)
Hence, by the converse of Thales theorem DE || BC
12. Resulting solid is a cuboid with length = 3(15) = 45 cm ; breadth = 15 cm ;
Height = 15 cm
TSA of the cuboid = 2 (lb + bh + hl)
= 2 ( 675 + 225 +675)

= 3150 cm2
13. Let n be any arbitrary positive odd integer.
On dividing n by 4, let m be the Quotient and r be the remainder. So, by Euclid’s division
lemma, we have
n = 4m + r, where m ≠ 0 and r < 4.
As m ≠ 0 and r < 4 and r is an integer, r can take values 0, 1, 2, 3.
n = 4m or n = 4m + 1 or n = 4m + 2 or n = 4m + 3
n = 4m + 1 or n = 4m + 3( since n is odd)
Thus, any positive odd integer is of the form (4m + 1) or (4m + 3), where m is some
integer.
14. Since and - are two zeroes of the polynomial
(x - ) (x + ) is a factor of the polynomial.

By long division method x4 + 3x3 - 20x2 - 6x + 36

= (x2 - 2) (x2 + 3x - 18)

= (x2 - 2) (x + 6) (x - 3)
The other zeroes of the Polynomial are -6,3. 1
15.

(x2- 5x + 4) +(x2 - 5x + 6)=

(x2 - 6x + 8)

6x2 – 30x +30 = 10x2 -60x +80

Material downloaded from myCBSEguide.com. 8 / 19


175
4x2 – 30x +50=0

2x2 – 15x +25 = 0


(x-5)(2x-5) = 0
x=5; x=5/2
16. 96,100, 104,.........
a = 96 , Sn = 168 ×12 = 2016 ; d = 4

Sn = (2×96 + (n – 1) 4)=2016

n2+47n-1008=0
(n+63)(n-16) =0
n=-63 is rejected; n =16
Therefore no of boys in the class = 16
17. The vertices of triangle are A(2 , ),

Therefore the coordinates of centroid are

18.

19.

Material downloaded from myCBSEguide.com. 9 / 19


176
= 2 cosec A
= RHS

OR

L.H.S

20. Let the side of the park be ‘a’ meter.

5 a2 = 720 a = 12m.
Perimeter of square = 48 m.
Perimeter of triangle = 48m.
Side of triangle = 16m.
Now Area of triangle

= = 64 m 2.
Cost of maintaining the park
= Rs. (10 64 )
= Rs. 1108.48
21.

Marks No.of students(fi)

0 – 10 7

10 - 20 14

20 - 30 13

30 - 40 12(f0)

40 - 50 20(f1)

Material downloaded from myCBSEguide.com. 10 / 19


177
50 - 60 11(f2)

60 - 70 15

70 - 80 8

TOTAL 68

Modal class is 40 – 50 because it has highest frequency

Mode =

OR

Marks (class) No. of students (Frequency) Cumulative Frequency

30-35 14 14

35-40 16 30

40-45 18 48

45-50 23 71

50-55 18 89

55-60 8 97

60-65 3 100

Here n = 100
Therefore , which lies in the class 45-50
l1 (The lower limit of the median class) = 45

c (The cumulative frequency of the class preceding the median class) = 48


f (The frequency of the Median class)= 23
h (The class size) = 5

Median

Material downloaded from myCBSEguide.com. 11 / 19


178
So, the median percentage of marks is 45.4
22. Let the number of blue balls be ‘x’.
Thus the sum of the possible outcomes = 6 +x
Now the sum of the favourable outcomes if the red balls are drawn = 6
P(E) =
Again the sum of the favourable outcomes if the blue balls are drawn = x
P(E) =
=2 ×

x=12
Hence the number of blue balls = 12.
23.

Since the lines intersect at (3, 3), there is a unique solution given by x=3, y = 3
Area of triangle ABC formed by lines with x - axis = ½ x 10 x 3 = 15sq. units
Area of triangle BDE formed by lines with y - a x is = ½ x 10 x 3 = 15 sq units
Ratio of these areas = 1 :1

OR

Material downloaded from myCBSEguide.com. 12 / 19


179
Let the speed of boat is x km/h in still water
and stream y km/h
According to question,

on solving eq. (i) and (ii) we get,

on solving eq. (iii) and (iv) we get,


x = 8km/h
y = 3km/h
24. Let the usual speed of plane be x km/hour

Time taken = hrs. with usual speed

Time taken after increasing speed = hrs

Given that

x2 + 250x - 750000 = 0
(x + 1000 ) ( x -750 ) = 0
x = 750 or -1000 (Rejected)
Usual speed of plane = 750km/h.
25. Given: ABC ~ DEF
To Prove:

Construction: Draw AG BC and DH EF


Proof:

Material downloaded from myCBSEguide.com. 13 / 19


180
Now in ABG and DEH, B = E
AGB = DHE(Each 90°)
AGB ~ DHE

But,

From (i) and (ii), we get,

Similarly, we can prove

Let the largest side of the larger triangle be x cm, then


(Using the theorem)

x = 36 cm

OR

Given: A such that

To prove: Triangle ABC is right angled at B


Construction: Construct a triangle DEF such that
and
Proof: is a right angledtriangle right angled at E [construction]

Material downloaded from myCBSEguide.com. 14 / 19


181
By Pythagoras theorem, we have

Thus, in and we have


[By Construction and (i)]

Hence, is a right triangle.


In

Now in
is right angled at K

26.

Material downloaded from myCBSEguide.com. 15 / 19


182
Since tangent is perpendicular to the radius of the circle
SPO = SRO = OQT = 90°
In right triangles OPS and ORS
OS = OS (common)
OP = OR (radii of circle)
OPS ORS(RHS congruence)
1 = 2
Similarly 3= 4
Now 1 + 2 + 3 + 4 =180°
2 + 3 =90°
SOT = 90°
27. DABC in which AB = 4 cm, BC = 6 cm and ABC = 60°
DA’BC’ is the required similar triangle.

28.

In right BAC,

Material downloaded from myCBSEguide.com. 16 / 19


183
AB = (100 + AD) x ---(i)

In right DBAD ,

AB = AD ........ (ii)
From (i) and (ii) we get
100 + AD = 3 AD
AD = 50 m
From (ii) AB = 50 m
= 50 1.732m
or, AB = 86.6 m
29. The Container is a frustum of cone h = 16cm, r = 8cm, R = 20cm

Volume of the container = ph ( R2 + Rr + r2 )

= 3.14 16 ((20)2 + 20(8) + (8)2) cm3

= 3.14 16 (400 + 160 +64) cm3

= ( 3.14 16 624 ) cm3

= (3.14 3328) cm3

= 10449.92 cm3
= 10.45 litres
Cost of milk = Rs (10.45 15) = Rs 156.75
Now, slant height of the frustum of cone = l

Total surface area of the container = pl ( R+r) +p r2

= 3.14 x 20 (20 + 8) + 3.14 (8)2 cm2

= 3.14 [ 20 x 28 + 64 ] cm2

= 3.14 x 624 = 1959.36 cm2


Cost of metal Used = Rs 1959.36 x
= Rs 19.5936 x 5

Material downloaded from myCBSEguide.com. 17 / 19


184
= Rs 97.968
= Rs 98 (Approx.)
30.

CLASS INTERVAL FREQUENCY CUMULATIVE FREQUENCY

0-5 7 7

5-10 10 17

10-15 x 17+x

15-20 13 30+x

20-25 y 30+x+y

25-30 10 40+x+y

30-35 14 54+x+y

35-40 9 63+x+y

Given n(total frequency ) = 100


100 = 63 + x + y
x + y = 37 ----------(1)
The median is 20.75 which lies in the class 20-25
So, median class is 20-25
l = 20
f = y
c.f = 30 + x
h = 5
Using formula,

Median = l +

20.75 = 20 +

3y = 400 - 20x
20x + 3y = 400 ------(2)
Solving (1) and (2), we get
x = 17

Material downloaded from myCBSEguide.com. 18 / 19


185
y = 20

OR

Monthly consumption of No. of consumers Class Mark


C.F FX
electricity (F) (X)

65-85 4 4 75 300

85-105 5 9 95 475

105-125 13 22 115 1495

125-145 20 42 135 2700

145-165 14 56 155 2670

165-185 8 64 175 1400

185-205 4 68 195 780

∑ƒx=9320

Now and this is in 125-145 class


Median class = 125-145
Here,

We know that

Hence, Median = 137

Again Mean

For mode, since the maximum frequency is 20 and this corresponds to the class 125-145
Here,

Thus, Median = 137, Mean = 137.05 and Mode = 135.76


The three measures are approximately the same in the class.

Material downloaded from myCBSEguide.com. 19 / 19


186
CBSE Class-X Mathematics
Sample Paper 11

General Instructions

i. All questions are compulsory.


ii. The question paper consists of 30 questions divided into four sections A, B,C and D.
iii. Section A contains 6 questions of 1 mark each, Section B contains 6 questions of 2 marks
each, Section C contains 10 questions of 3 marks each and Section D contains 8 questions
of 4 marks each.
iv. Use of Calculator and log tables is not permitted.

1. Find the value of k for which the pair of linear equations and
has no solution
2. for what value of p, are 2p + 1, 13, 5p-3 three consecutive terms of an A.P?
3. Without using trigonometry table, evaluate

4. In figure, , BC = 7.5 cm, AM = 4cm and MC = 2 cm. Find the length BN

5. The two tangents from an external point P to a circle with centre O and PA and PB. If
what is the value of ?
6. A pair of dice is thrown once. Find the probability of getting the same number on each
dice.
7. Prove that is an irrational number.
8. Solve the equations graphically:

What is the area of the triangle formed by the two lines and the line y = 0 ?
9. Find the roots of the following equation

Material downloaded from myCBSEguide.com. 1 / 26


187
10. Prove that :
11. The angle of elevation of the top of a building from the foot of a tower is and angle of
elevation of top of the tower from the foot of the building is . If the tower is 50 m
high. Find the height of the building.
12. The dimensions of a metallic cuboid are 100 cm 80 cm 64 cm. It is melted and recast
into a cube. Find the surface area of the cube.
13. A person on tour has Rs. 4200 for his expense. If he extends his tour for 3 days , he has to
cut down his daily expense by RS. 70. Find the duration of the tour.
14. The first and last term of an AP are 8 and 350 respectively. If its common difference is 9,
how many terms are there and what is their sum?
15. Prove that the ratio of areas of two similar triangles is equal to the ratio of the square of
their corresponding sides.
16. Point P divides the line segment joining the points A (2,1) and B (5, -8) such that
If P lies on the line , find the value of k.

OR

Show that the points and are the vertices of a


square.
17. From the top of a 7m high building, the angle of elevation of the top of a tower is and
angle of depression of the foot of the tower is Find the height of the tower.

OR

A girl who is 1.2 m tall, spots a balloon moving with the wind in a horizontal line at a
height of 88.2 m from the ground. The angle of elevation of the balloon from the eye of
the girl at any instant is . After sometime, the angle of elevation reduces to . Find
the distance travelled by the balloon during the interval.
18. In figure, a triangle ABC is drawn to circumscribe a circle of radius 2 cm such that the
segment BD and DC into which BC is divided by the point of contact D are the lengths 4cm
and 3cm respectively. If area of ABC = 21 , then find the lengths of sides AB and AC.

Material downloaded from myCBSEguide.com. 2 / 26


188
19. Draw a triangle ABC with BC = 7cm, and . then construct another
triangle whose sides are times the corresponding sides of ABC.

OR

Draw a circle of radius 2.3 cm take a point P on it. Without using the centre of the circle.
Draw tangent to it at P.
20. The mid points of sides of a triangle are (3,4) , (4,1) and (2,0). Find the coordinate of
vertices of the triangle.
21. Weekly income of 600 families is given below.

Income in Rs. Frequency

0 - 10000 250

1000 2000 190

2000 3000 100

3000 4000 40

4000 5000 15

5000 6000 5

Find the median.

OR

The following tables gives production yield per hectare of wheat of 100 farms of village:

Production Yeild (in hr.) No. of Farms

Material downloaded from myCBSEguide.com. 3 / 26


189
50 - 55 2

55 - 60 8

60 - 65 12

65 - 70 24

70 - 75 38

75 - 80 16

Change the distribution to a more than type distribution and draw its Ogive.

22. All kings, queens and aces are removed from a pack of 52 cards. The remaining cards are
well shuffled and then a card id drawn from it. Find the probability that the drawn card
is
i. a black face card
ii. a red card.
23. Show that only one of the number n, n+2 and n+4 is divisible by 3.

OR

Use Euclid Division Lemma to show that cube of any positive integer is either of the form
9m, (9m + 1) or (9m + 8).
24. Obtain all other zeroes of the polynomial , if two of its zeroes
are and .
25. Solve the following pair of equations for and

OR

If in a rectangle the length is increased and breadth is decreased by 2 units each, the area
is reduced by 28 square units, and if the length is reduced by 1 unit and breadth is
increased by 2 units, the area increased by 33 square units. Find the dimensions of the
rectangle.
26. In figure AB units, CD units and units, prove that

Material downloaded from myCBSEguide.com. 4 / 26


190
27. Prove that

28. ABC is a right triangle, right angled at A. Find the area of shaded region if AB = 6cm, BC =
10cm and O is the centre of incircle of ABC. (take )

29. A gulab jamun , when ready for eating, contains sugar syrup of about 30% of its volume.
Find approximately how much syrup would be found in 45 such gulab jamuns, each
shaped like a cylinder with two hemispherical ends, if the complete length of each of
them is 5cm and its diameter is 2.8 cm.
30. Draw ‘less than Ogive’ and ‘more than Ogive’ for the following distribution and hence
find its median.

Class Frequency

20 - 30 10

30 - 40 8

40 - 30 12

50 - 60 24

60 - 70 6

Material downloaded from myCBSEguide.com. 5 / 26


191
70 - 60 25

80 - 90 15

OR

Find the mean, mode and median for the following data.

Classes Frequency

5 - 15 2

15 - 25 3

25 - 35 5

35 - 45 7

45 - 55 4

55 - 65 2

65 - 75 2

Material downloaded from myCBSEguide.com. 6 / 26


192
CBSE Class-X Mathematics
Sample Paper 11
Solutions

1. Since pair of equations has no solution


Then,

i.e.

2. 2p+1, 13 ,5p-3 are consecutive terms of an A.P

= 5p - 3 - 13
13 - 2p - 1 = 5p - 16
12 - 2p = 5p - 16
12 + 16 = 5p + 2p
28 = 7p p = 4
3.

4. In MN AB

[ Let x = BN]

3x = 15 x = 5
Hence BN = 5 cm.
5. PA and PB are tangents to the circle

Material downloaded from myCBSEguide.com. 7 / 26


193
[Tangent makes an angle of with the radius at the point of contact ]
In quadrilateral OAPB,

[Angle sum property of a quadrilateral]

6. Total number of outcomes when a pair of dice is thrown is 36


Same number on each dice i.e. (1,1) , (2,2) , (3,3) , (4,4) , (5,5) , (6,6)
Number of ways of getting the same number on each dice = 6.
Required probability =
7. Let if possible is a rational number.

From we notice
LHS is an irrational number and RHS is rational number, which is not possible. Hence,
our supposition is wrong. Hence, is an irrational number.
8.

1 0 2

0 2 -2

Material downloaded from myCBSEguide.com. 8 / 26


194
From

0 -4 2

2 0 3

From

From graph, we observe that solution of equation is (0,2)


Area
= 5 square units
9.

x2 - 3x - 28 = -30

x2 - 3x + 2 = 0
(x - 2)(x - 1)=0
x = 2,1
Required roots are 2, 1
10. Taking LHS

Material downloaded from myCBSEguide.com. 9 / 26


195
=

=RHS
11.

Let TP be height of tower of 50m


,
Let distance AP between building and tower be .
To find : AB, height of building
In right
In right

AB

AB = 16.67m
Hence height of the building is 16.67 m
12. Dimensions of the metallic cuboid are
100 cm 80 cm 64 cm
100 80 64
(Where a is side of cube)

a = 80 cm

Material downloaded from myCBSEguide.com. 10 / 26


196
Surface area of cube


13. Number of days for a tour = x
Daily expense = y
A.T.Q.

If tour be extended for 3 days


New number of days = x + 3
Daily expense = y - 70
Now (x + 3)(y - 70) = 4200
xy - 70x + 3y -210 = 4200
-70x + 3y - 210 = 0 [From (i)]
Put

x2 - 180 + 3x = 0

x2 + 3x - 180 = 0
(x + 15)(x - 12) = 0
x = 12, x = -15 (rejected)
Duration of Tours = 12 Hours
14. Here a = 8, l =350, d=9
From formula, l= an = a + (n - 1) d, we get

a + ( n-1) d=350
8 + (n - 1)9 = 350
(n - 1)9 = 350 - 8
(n - 1)9 = 342
n - 1 =
n - 1 = 38
n = 38 + 1

Material downloaded from myCBSEguide.com. 11 / 26


197
From formula, we get

15. Given :

To prove :
Construction : Draw AM BC and DK EF
Proof : ABC

Also ABM and DEK

(given) ( angles of similar s)


(Construction)
By AA Rule

Equating (i) and (ii)

So, Hence proved


16. P is the point of intersection of line segment AB and line 2x - y + k = 0.
Here,
3 AP = AP + PB
2 AP = PB
AP : PB = 1 : 2
P divides the line segment joining A (2,1) and B(5, -8) in ratio 1:2

Material downloaded from myCBSEguide.com. 12 / 26


198
Coordinates of point P are

i.e. p(3, -2)

As point P lies on the line 2x - y + k = 0.


6 + 2 + k = 0 6 = -8

OR

Diagonal

Diagonal
Hence proved.
17. AB is a building of height 7 m and CD is tower of height h m.

Material downloaded from myCBSEguide.com. 13 / 26


199
AB = ED = 7m
and CE = (h-7)m
Let BD = AE = x m
In right AED,

In right AEC,

h = 28m
Height of the tower is 28 m.

OR

In right

In right

Material downloaded from myCBSEguide.com. 14 / 26


200
18. Let AE = AF = x
Length of tangents from an external point is equal.

ar BOC =
ar AOB =
ar AOC =

[from (i)]

21x + 7x - 49 = 0

2x2 + 7x - 49 = 0

2x2 + 14x - 7x -49 = 0


2x(x + 7) - 7(x + 7) = 0
(2x - 7)(x + 7) = 0
[rejected]
length of side AB = 4+ 3.5 = 7.5 cm and AC =3 + 3.5 = 6.5 cm
19.

Material downloaded from myCBSEguide.com. 15 / 26


201
OR

Steps of construction:

1. Draw a circle of radius 2.3 cm and take a point P on it.


2. Draw chord PQ.
3. Mark a point R in the major arc QP.
4. Join PR and RQ.
5. Draw
6. Produce to as shown in figure, then is the required tangent at the point P.
20. Let A(x1 , y1), B(x2, y2) and C(x3 , y3) are the vertices of ABC.

(3,4), (4,1),(2,0) are mid points of sides AB,BC,CA


As (3,4) is mid point of AB
x1 + x2 = 6 ....(i)

y1 + y2 = 8 ...(ii)

Material downloaded from myCBSEguide.com. 16 / 26


202
As (4,1) are mid points of BC
x2 + x3 = 4 ...(iii)

y2 + y3 = 2... (iv)
As (2,0) are mid points of AC
x1 + x3 = 4 ...(v)

y1 + y3 = 0 ..(vi)

Adding (i), (iii), (v), we get


2 (x1 + x2 + x3) = 18

x1 + x2 + x3 = 9

as x1 + x2 = 6 [from (i)]

x3 = 3

x1 = 1, x2 = 5

Similarly, y1 = 3, y2 = 5, y3 = -3

Coordinate of vertices of are A(1, 3), B(5, 5), C(3, -3)


21.

Income in Rs. Number of families (f) c. f

0 - 1000 250 250

1000 - 2000 190 440

2000 - 3000 100 540

3000 - 4000 40 580

4000 - 5000 15 595

5000 - 6000 5 600

n 600 = 300
Median class = 1000 - 2000
l = 1000, c.f.=250, f=190, h=1000

Median

OR

Material downloaded from myCBSEguide.com. 17 / 26


203
More than type Ogive

Production yield (Kg/ha) C.F

More than or equal to 50 100

More than or equal to 55 98

More than or equal to 60 90

More than or equal to 65 78

More than or equal to 70 54

More than or equal to 75 16

Now, draw the Ogive by plotting the points (50, 100), (55, 98), (60, 90), (65, 78), (70, 54), (75,
16)
22. Number of kings = 4, number of queens =4,

number of aces = 4
After removing all kings, queens and aces,
number of cards left = 52 – 12 = 40 (i) Number of black face cards in the remaining cards (
2 jacks) = 2
Probability of black face card
(ii) Out of 12 cards removed, 6 are of red colour.
Number of red- coloured cards left 26 – 6 = 20
Number of ways of drawing a red card = 20
Probability of getting a red card
23. Let n = 3k ; 3k+1 or 3k +2
i. When n = 3k n is divisible by 3

Material downloaded from myCBSEguide.com. 18 / 26


204
n+ 2 = 3k +2
n + 2 is not divisible by 3
and n + 4 = 3k + 4 = 3k + 3 + 1
= 3(k +1) + 1
n + 4 is not divisible by 3.
ii. When n = 3k + 1
n is not divisible by 3
n + 2 = (3k + 1) + 2 = 3k + 3
= 3 (k + 1)
n + 2 is divisible by 3
n + 4 = 3k + 1 + 4 = 3k + 5
= 3k + 3 + 2 = 3(k +1)+2
∴ n + 4 is not divisible by 3
iii. When n = 3k + 2, n is not divisible by 3
n + 2 = (3k +2) +2 = 3k + 4
= 3(k +1) +1
n + 2 is not divisible by 3
n + 4 = (3k +2) + 4
= 3k + 6 = 3(k+2)
n + 4 is divisible by 3.
Only one of the numbers n , n + 2 and n + 4 is divisible by 3.

OR

Let a = 3q + r

a = 3q; then a3 = 27q3=9m ; where m=3q3

when a=3q+1 ; then a3 =27q3+27q2+9q+1

= 9 (3q3+3q2+q)+1 = 9m+8 (where m = 3q3 + 3q2+q)

when a=3q+2 ; then a3= (3q+2)2

= 27q3+54q2+36q+8

= 9(3q3+6q2+4q)+8

= 9m+8 (where m = 3q3+6q2+4q)


Hence, cubes of any positive integer is either of the form9m,( 9m+1 ) or( 9m+8 ).

Material downloaded from myCBSEguide.com. 19 / 26


205
24.
are the zeroes of
is a factor of
is factor of
For other factor

other factor of p(x) is x2 - 3x + 2=0

For other zeroes, x2 - 3x + 2=0


(x - 2)(x - 1)
x = 2, x = 1
Other zeroes are 1 and 2.
25.
Put and

also ; put and

We have

Equation (i) - 2 x equation (ii)

x - y = 5 ..(iii)

Material downloaded from myCBSEguide.com. 20 / 26


206
Put

Solving (iii) and (iv) for x and y


We have

OR

Let the length and breadth of a rectangle be x and y meters.


According to question,
Area = xy
(x + 2) (y - 2) = xy - 28
or 2x - 2y = 24
or x - y = 12 ...(i)
and (x - 1) (y + 2) = xy + 33
2x - y = 33 ....(ii)
on subtracting eq (ii) - (i), we get,
x = 21 and 21 - y = 12
so y = 21 - 12
y = 9
length =21m, breadth = 9m
26.

Let BQ = units, DQ b units

and

Material downloaded from myCBSEguide.com. 21 / 26


207
Similarly

Also

From (i) and (ii)

(Hence Proved )
27.

Taking LHS

Material downloaded from myCBSEguide.com. 22 / 26


208
=

=
=
= = RHS
28.

Since and OP, OQ are radius through contact point must be on tangents
lines.
Therefore, OPAQ is a square.
Let OP AP = AQ = OQ
Now in ABC ,

i.e. 36 + AC2 = 100

AC2 = 64 Ac = 8
BR = PB = 6 - x
CQ = 8 - x = CR
[tangent from an external point]
Now again BC = CR + BR
10 = 8 - x + 6 - x
10 = 14 - 2x 2x = 4
x = 2
Area of shaded portion = Area of triangle - Area of circle

29.

Material downloaded from myCBSEguide.com. 23 / 26


209
Radius of the hemispherical part = 1.4 cm
Length of the cylindrical part = 5 - (2 x 1.4) = 2.2cm
Volume of 1 gulab jamun = volume of two hemispherical ends + Volume of cylindrical
part

Volume of 45 gulab jamuns


Volume of syrup found in 45 gulab jamuns

30. Solution : table for ‘less than Ogive’ and ‘more than Ogive’

For ‘less than’ Ogive For ‘more than’ Ogive


C.I.
C.I. (less than) c.f. Point C.I. (more than) c.f. Point

20- 30 10 30 10 (30,10) 20 100 (20,100)

30 - 40 8 40 18 (40,18) 30 90 (30,90)

40 - 50 12 50 30 (50,30) 40 82 (40,82)

50 60 24 60 54 (60,54) 50 70 (50,70)

60 - 70 6 70 60 (70,60) 60 46 (60,46)

70 - 80 25 80 85 (80,85) 70 40 (70,40)

80 - 90 15 90 100 (90,100) 80 15 (80,15)

‘Less than’ Ogive and ‘more than’ Ogive.

Material downloaded from myCBSEguide.com. 24 / 26


210
We notice both the curves intersect at (58.3, 50)
Median = 58.3

OR

We have,

class Mid- value(xi) (fi) fiu i c.f

5-15 10 2 -3 -6 2

15-25 20 3 -2 -6 5

25-35 30 5 -1 -5 10

35-45 40 7 0 0 17

45-55 50 4 1 4 21

55-65 60 2 2 4 23

65-75 70 2 3 6 25

Material downloaded from myCBSEguide.com. 25 / 26


211

Let assumed mean ‘a’ = 40, Here h=10

Mean

= 40 - 1.2 = 38.8
Since, maximum frequency = 7
Modal class = 35-45
Here, l = 35, f1 = 7, f0 = 5, f2 = 4

We know that

Mode

= 35 + 4 = 39
Since which lies in 35-45 class
Here,

Median =

= 35 + 3.6 nearly = 38.6 nearly

Material downloaded from myCBSEguide.com. 26 / 26


212
CBSE Class 10 Mathematics
Sample Paper 12

Time: 3 hours Max Marks: 80


General Instructions

i. All questions are compulsory.


ii. The question paper consists of 30 questions, divided into four sections-A, B,C & D
iii. Section A contains 6 questions of 1 mark each, section B contains 6 questions of 2 marks
each, Section C contains 10 questions of 3 marks each and Section D contains 8 questions
of 4 marks each.
iv. Use of calculator is not permitted.

SECTION-A

1. If one zero of quadratic polynomial x2 - x - (2 + 2k) is - 4, find the value of K.


2. If & what is the value of ?
3. A ladder 15m long just reaches the top of a vertical wall. If the ladder makes an angle of
60° with the wall. Find the height of the wall.

4. Total surface area of a cube is 216 cm2. what will be its volume?
5. If the probability of winning a game is 0.995 then what will be the probability of losing a
game?
6. What is the ordinate of a point on x axis?

SECTION-B

7. Find the sum of first 23 terms of an A.P. 7, , 14 ‫ و‬.........


8. A card is drawn at random from a well shuffled deck of 52 cards, find the probability that
card drawn is neither a red card nor a queen.
9. If the points A(4,3) & B(x, 5) are on the circle with the center 0(2,3), find the value of x.
10. Simplify
11. Prove that
12. If H.C.F. (306, 144) = 18, find the LCM.

Material downloaded from myCBSEguide.com. 1 / 17


213
SECTION-C

13. Prove that is an irrational number.

14. If polynomial f(x) = x4-3x3 -x2 + 9x – 6 has two zeroes as find all zeroes of the
polynomial

Or

If one zero of 2x2 + px - 15 is -5 and zeroes of p(x2 + x) + k are equal to each other, find the
value of p & k.
15. For what value of a & b, the following pair of linear equation represents the coincident
lines
2x +3y = 7
a(x + y)-b (x -y) =3a + b - 2

16. Find the value of P such that the quadratic equation (p-12)x2 - 2(p - 12)x + 2 = 0 has equal
roots.

Or

Solve the equation for x:

17. Find the sum of all 3 digit nos. which leaves the same remainder 2 when divided by 5.
18. In the given figure, is right angled at CIf BC = a, CA = b, AB = C & P is the length of
perpendicular drawn from C to AB then prove that-

i. cp = ab
ii.

19. In the given figure, two tangents PQ & PR are drawn to a circle with centre O from an

Material downloaded from myCBSEguide.com. 2 / 17


214
external point P. Prove that

20. The length of a rope by which a cow is tethered to one end, of a corner of rectangle
increased from 16m to 23m. How much additional area can the cow graze now?

21. A solid wooden toy is in the form of a cone mounted on a hemisphere. If the radii of
hemisphere and base of cone are 4.2 cm each and the total height of toy is 10.2 cm, find
the volume of wood used in the toy. Also, find the total surface area of toy.
22. The mean of following distribution is 50, find the values of f1, f2 –

Class interval 0-20 20-40 40-60 60-80 80-100 Total

Frequency 17 f1 32 f2 19 120

SECTION-D

23. From a window, 60m high above the ground of a house in a street, the angles of elevation
& depression of the top & foot of another house on the opposite side of the street are 60° &
45° respectively. Show that the height of the opposite house is metres.

Or

Prove that .

24. The taxi charges in a city comprise of a fixed charge together with the charge for the
distance covered. For a journey of 10 km, the charges paid are Rs. 75 and for a journey of
15 km, the charges paid are Rs. 110. What will a person have to pay for travelling a
distance of 25 km?

Or

If two pipes functions simultaneously, a reservoir will be filled in 12 hours. First pipe fills

Material downloaded from myCBSEguide.com. 3 / 17


215
the reservoir 10 hours faster than the second pipe. How many hours will the second pipe
take to fill the reservoir.
25. Construct in which AB = 5 cm, & the altitude CD = 3 cm then construct
another whose sides are times the corresponding sides of .
26. A bucket is in the form of a frustrum of a cone of height is 30 cm with radii of its lower
and upper ends as 10 cm and 20 cm respectively. Find the capacity of the bucket. Also,
find the cost of milk which can completely fill the container, at the rate of Rs. 25 per liter
(Take T = 3.14)
27. A jar contains 24 marbles, some are green and others are blue. If a marble is drawn at
random from the jar, the probability that it is green is 2/3. Find the number of blue
marbles.
28. State and prove pythagoras theorem

Or

Prove that the tangent at any point of a circle is perpendicular to the radius through the
point of contact.
29. The distribution given below shows the marks of 100 students of a class-

Marks No. of Students

0-5 4

5-10 6

10-15 10

15-20 10

20-25 25

25-30 22

30-35 18

35-40 5

Draw a less than type and a more than type ogive from the given data. Hence obtain the
median marks from the graph.
30. Find the area of triangle formed by joining the midpoints of the sides of the triangle
whose vertices (0, + 1) (2, 1) & (0.3), find the ratio of this area to the area of given triangle.

Material downloaded from myCBSEguide.com. 4 / 17


216
CBSE Class 10 Mathematics
Sample Paper 12
Solution

SECTION -A

1. One zero of

x2 - x - (2 + 2k) is -4

(-4)2 - (-4) - (2 + 2k) = 0


16 + 4 - 2 - 2k = 0
18 - 2k = 0
-2k = -18
k = 9
2.

= 180 - (180o + 60o)


= 180 - 140

= 40o
3. In

Material downloaded from myCBSEguide.com. 5 / 17


217
Height of the wall

4. Total surface area of cube = 216 cm2

Volume of cube = a3 = 63 = 216 cm3


5. P(E) = 0.995
P(E) = 1 - P(E)
= 1 - 0.995
= 0.005
6. The ordinate of a point on x-axis is 0.
7. A.P. is 7, , 14, ........

8. n(s) = 52
Red Cards = 26
Black Queens = 02
Let A denotes the no. of cards neither Red nor black card queens
n(A) = 52 - 26 - 02 = 24

9. 0 is the midpt. of AB.

Material downloaded from myCBSEguide.com. 6 / 17


218
x + 4 = 4
x = 4 - 4
x = 0
10.

11. L.H.S.

R.H.S.

LHS = RHS
12. N1 = 306, N2 = 144

HCF = 18

= 2448

13. To Prove is an irrational we will first prove is an irrational number.


Let if possible, is a rational number
p & q are co-prime integers.
Squaring both sides

3q2 = p2 .... (1)

p2 is divisible by 3

Material downloaded from myCBSEguide.com. 7 / 17


219
p is divisible by 3

If a prime number divides a2 then it also divides a ........ (A)


p = 3m ....... (2)
Substituting (2) in (1)

3q2 = (3m)2

3q2 = 9m2

q2 = 3m2

3 divides q2
3 divides q using A
q = 3n ..... (3)
2 & 3 3 divides both p and q which contradicts the fact that p and q are co-prime
Our assumption is false is an irrational number.

is an irrational number as product of rational & irrational number is an


irrational number.

14. f(x) = x4 - 3x3 - x2 + 9x - 6


are two zeroes of f(x)
will divide f(x)

x2 - 3 is a factor of f(x)

f(x) = (x2 - 3) (x2 - 3x + 2)

Zero of (x) =

Material downloaded from myCBSEguide.com. 8 / 17


220
Or

Once zero 2x2 + px - 15 is -5

2(-5)2 + p(-5) - 15 = 0
50 - 5p - 15 = 0
-5p = -35
p = 7

p(x2 + x) + k

= 7(x2 + x) + k

= 7x2 + 7x + k

4k = 7

15. 2x + 3y = 7 ....... (1)


a(x + y) - b (x - y) = 3a + b - 2
or x(a - b) + y(a + b) = 3a + b - 1 ...... (2)
for coincident lines

If
7a - 7b = 6a + 2b - 4
a - 9b = -4 ..... (3)
If
7a + 7b = 9a + 3b - 6
-2a + 4b = -6
a - 2b = 3 ....4

Material downloaded from myCBSEguide.com. 9 / 17


221
Putting in ..... 3

a = 4 + 9
a = 5

16. (p - 12)x2 - 2(p - 12)x + 2 = 0


a = p - 12, b = -2(p - 12), c = 2
for equal roots
D = 0

b2 - 4a c = 0

4(p - 12)2 - 8(p - 12) = 0

(p - 12)2 - 2(p - 12) = 0


(p - 12) (p - 12 -2) = 0
(p - 12) (p - 14) = 0
p = 12, 14

Or

2x2 + 2 = 17x - 34

2x2 - 17x + 36 = 0

2x2 - 9x - 8x + 36 = 0
x(2x - 9) - 4(2x - 9) = 0
(2x - 9) (x - 4) = 0

17. Three digit nos, divisible 5 leaving remainder 2 in each case are-

Material downloaded from myCBSEguide.com. 10 / 17


222
102, 107, ......, 997
an = 997, a = 102, d = 107 - 102 = 5

102 + (n - 1) 5 = 997
102 + 5n - 5 = 997
5n + 97 = 997
5n=997 - 97
5n = 900

= 98910
18.

i. ar
..... (1)
Also air
.... (2)
from 1 & 2

ii.

19. Let
We know that PQ = PR (The length of the tangents drawn from an external point are

Material downloaded from myCBSEguide.com. 11 / 17


223
equal)
PQR is an isosceles triangle

20. Addition area grazed

21. r = 4.2 cm
height of cone = 10.2 - 4.2
= 6cm

= 7.32 cm
Volume of wood

Material downloaded from myCBSEguide.com. 12 / 17


224
= 266.112 cm3
The surface area of toy

= 96.36 + 110.88

= 207.24 cm2
22.

Class interval x f fx

0 - 20 10 17 170

20 - 40 30 f1 30f1

40 - 60 50 32 1600

60 - 80 70 f2 70f2

80 - 100 90 19 1710

120 3480+30f1+70f2

Mean = 50

3480 + 30f1 + 70f2 = 6000

348 + 3f1 + 7f2 = 600

3f1 + 7f2 = 252 .... (1)

Also 17 + f1 + 32 + f2 + 19 = 120

f1 + f2 = 52 ..... (2)

Multiplying equation 1 by 1 & 2 by 3

f1 + 24 = 52

f1 = 52 - 24 = 28

Material downloaded from myCBSEguide.com. 13 / 17


225
23. In

In

Height of wall = 60

Or

LHS

RHS

Material downloaded from myCBSEguide.com. 14 / 17


226
LHS = RHS

24. Let fixed fare = Rs. x


fare per km = Rs. y

y = 7

Put y = 7m - qn D

x = 75 - 10
x = 5
Fixed fare = x = Rs. 5
Fare for 1 km = y = Rs. 7
Fare for 25 km = x + 25y

= Rs. 180

Or

Let time taken by small llameher pipe = n hr


time taken by b1q llameler pipe = (n - 10) hr and

n2 - 10n = 24n -120

n2 - 34 + 120 = 0

n2 - 30n - 4n + 120 = 0
n(n - 30) - 4(n - 30) = 0
(n - 30) (n - 4) = 0
n = 30, n = 4
time taken by small diameter pipe = 30 hrs
time taken by diameter pipe = 20 hrs

Material downloaded from myCBSEguide.com. 15 / 17


227
25.

26. h = 30 cnm r = 10, R = 20 cm


vol of by neat

Cost of milk

27. Total marbles = 24


Let green marbles = x
blue marbles = 24 - x
p(green marbles)

blue marbles = 24 - 16 = 8

28.

29.

Less than type More than type

Mark No. of students Marks No. of students

Less than 5 4 More than 0 100

Less than 10 10 More than 5 96

Less than 15 20 More than 10 90

Less than 20 30 More than 15 80

Less than 25 55 More than 25 70

Less than 30 77 More than 30 45

Less than 35 95 More than 30 23

Less than 40 100 More than 35 5

Material downloaded from myCBSEguide.com. 16 / 17


228
30.

= 2 sq. units

= 1:4

Material downloaded from myCBSEguide.com. 17 / 17


229

S-ar putea să vă placă și